Download as pdf or txt
Download as pdf or txt
You are on page 1of 182

!

!
!
!
!
!
MEDI2011!!
Pathology!!
Tutorials!!
!
!
GUIDE TO EXAMINATION OF SPECIMENS DEMONSTRATING COMMON PATHOLOGICAL CONDITIONS

1. WHAT AM I LOOKING AT?


Note any given information including age, sex and clinical history
Identify any normal structure and the aspect it presents, e.g. the specimen may be
a spleen
part of a lung sectioned vertically
portion of a heart, sectioned to show left ventricle and left atrium

2. LOOK FOR NORMAL TISSUE AND ADJACENT ABNORMAL TISSUE

When mounting autopsy specimens an attempt is made to show normal adjacent tissue. This may not be
possible with specimens excised surgically.

3. DESCRIBE THE ABNORMALITIES IN TERMS OF FORM, TEXTURE AND COLOUR


Form
size
shape
number (single, multiple)
symmetry or asymmetry
Texture
smooth, rough
fine, coarse, spongy
glossy, dull
opaque, translucent, transparent
Colour - colour of specimens may be modified by fixation
form of each area of colour including margins and central parts
light, dark
homogeneous or patchy
showing patterns or resemblance to familiar things

4. LOOK AT ALL PARTS OF THE SPECIMEN


Front, back, sides, top and bottom
Anatomical structures, for example
Heart – inspect:
pericardium
myocardium
endocardium
each chamber
each valve
coronary arteries
Lung – inspect:
pleura and pleural surface of lung
cut surface of lung
air passages from hilum to periphery
blood vessels from hilum to periphery
hilar lymph nodes
Integrated Pathology Learning Centre 8.03.2013
GUIDE TO EXAMINATION OF SPECIMENS DEMONSTRATING COMMON PATHOLOGICAL CONDITIONS

5. MAKE A DIAGNOSIS
Specify:
the anatomical part/parts affected
the possible pathological process/processes involved
the possible primary cause/causes
the physiological or biochemical changes that may have occurred e.g. endocrine tumours may/may
not be hormonally active

IN ALL CASES CONSIDER THE FOLLOWING POINTS:


If multiple lesions are present try to relate them causally.
When a major lesion is identified, look for evidence of its causes as well as evidence of its possible
complications.
Try to assess the stage in the evolution of the pathological process/processes shown by the
specimen.
Multiple lesions in a specimen may be unrelated causally.

Integrated Pathology Learning Centre 8.03.2013


Pathology of lymphoma 2013
Page 1
MBBS2 Tutorial – Pathology of Lymphoma

Learning objectives:

Understand lymphoma, common types


Pathology of spleen and thymus

Lymphoid neoplasms
General points:
1. Gene mutations – either loss or gain in function
2. Oncoproteins – block normal maturation
3. Protooncogenes – often activated in lymphoid cells during antigen stimulation of germinal centres
leading to tumours of germinal center cell origin
4. Inherited factors – Downs Syndrome has increased risk of developing leukaemia
5. Viruses – HTLV1, EBV and KSV are associated with specific types of lymphoma
6. Chronic immune system stimulation – H pylori and MALT lymphoma
7. Iatrogenic – radiation treatment
8. Social factors – smokers have increased risk of AML

Definitions : Leukaemia
Lymphoma

Histology review – lymph node (histology and function)

Normal lymph node


architecture.

Leukaemia – To be discussed in next tutorial

WHO classification of lymphoma

Hogkins
lymphoma

Lymphoma

B cell ) further classified


NHL ) on degree of
T cell ) differentiation

Origin of lymphoid malignancy - B cell neoplasms – bone marrow and lymph nodes
- T cell noeplasma – thymus and lymph nodes
Pathology of lymphoma 2013
Page 2

Reproduced from Robbins


& Cotran Pathologic
Basis of Disease 7th Ed.
2005

Diagnosis of lymphoma requires histology.


Reactive lymphadenopathy leads to a polyclonal lymphoid proliferation whereas lymphomas lead to a
monoclonal lymphoid proliferation. Antigen receptor gene analysis uses this to differentiate reactive from
malignant condition.

80-85% of lymphomas are of B cell origin

Immunohistochemistry – B and T cell markers

Lymph node stained with B


cell marker (left) and T cell
marker (right)
B cells are in the germinal
centres while T cells are in the
peri follicular mantle zone.

Flow cytometry
Two broad groups – Hodgkins and NHL (clinically important as treatment different)
- highlight differences esp spread and staging

NHL
Discuss common types
Pathology of lymphoma 2013
Page 3
Hodgkins lymphoma
Types
B symptoms
Importance of staging

Spleen
Histology
Splenomegaly
Infarcts
Rupture
Neoplasia

Thymus
Histology

Neonatal thymus showing the


cortex and medulla.

Hyperplasia
Thymoma
Pathology of lymphoma 2013
Page 4
PATHOLOGY OF LYMPHOMA
Demonstration pots
Royal Brisbane Clinical School

Abdominal swelling from M/61 yrs


__________________________________
__________________________________
__________________________________
__________________________________
__________________________________
__________________________________

Abdominal mass F/55 yrs


________________________________________
________________________________________
________________________________________
________________________________________
________________________________________
Pathology of lymphoma 2013
Page 5

Abdominal mass M/46yrs


____________________________________________________
____________________________________________________
____________________________________________________
____________________________________________________
____________________________________________________

46 yrs F Spleen
________________________________________________________
________________________________________________________
________________________________________________________
________________________________________________________
_______________________________________________________

49 yrs/M with chronic valvular heart disease. Spleen


__________________________________________________
__________________________________________________
__________________________________________________
__________________________________________________
__________________________________________________
________________________________________________
Pathology of lymphoma 2013
Page 6

Incidental finding in the intestine M/44yrs


______________________________________________
______________________________________________
______________________________________________
_____________________________________________

Mediastinal organ (Normal) from baby 6 weeks old


__________________________________________________
__________________________________________________
__________________________________________________
__________________________________________________
Pathology of lymphoma 2013
Page 7

PATHOLOGY OF LYMPHOMA
Demonstration Pots
PA/Mater Clinical School

Small intestine & Mesentery mass


___________________________________________
___________________________________________
___________________________________________
___________________________________________
___________________________________________
___________________________________________

Mediastinal mass
_______________________________________________
_______________________________________________
_______________________________________________
______________________________________________

Splenomegaly
____________________________________________________
____________________________________________________
____________________________________________________
____________________________________________________
____________________________________________________
____________________________________________________
____________________________________________________
Pathology of lymphoma 2013
Page 8

46 yrs/M with Abdominal pain & shoulder pain 6 days


following a road traffic accident
_____________________________________________________
_____________________________________________________
_____________________________________________________
_____________________________________________________
_____________________________________________________

Patient with cirrhosis. Removed spleen


________________________________________________
________________________________________________
________________________________________________
________________________________________________
________________________________________________

33 yrs/M with Splenomegaly (Storage disease)


-
__________________________________________
__________________________________________
__________________________________________
__________________________________________
Pathology of lymphoma 2013
Page 9
PATHOLOGY OF LYMPHOMA
Demonstration Pots
Ipswich Clinical School

Small intestine & Mesentery mass


____________________________________
____________________________________
____________________________________
____________________________________
____________________________________

Lesion in the spleen.


____________________________________________
____________________________________________
____________________________________________
____________________________________________
____________________________________________
____________________________________________
____________________________________________

69 yrs/F axillary swelling.


__________________________________________________
__________________________________________________
__________________________________________________
__________________________________________________
__________________________________________________
__________________________________________________
Pathology of lymphoma 2013
Page 10

Normal spleen from an adult woman.


_____________________________________________
_____________________________________________
_____________________________________________
_____________________________________________

Splenomegaly
________________________________________________
________________________________________________
________________________________________________
________________________________________________
________________________________________________

M/75 yrs, Mediastinal mass.


___________________________________________
___________________________________________
___________________________________________
___________________________________________
Pathology of bone marrow/metabolic disease 2013
Page 1

MBBS2 Tutorial Outline – Pathology of Bone Marrow and Metabolic Bone Disease

Learning Objectives:
Bone marrow diseases
Bone and calcium metabolism and related diseases

Types of Marrow: Yellow (inactive)


Red (active)

Ways to examine bone marrow – aspirate, trephine

Normal bone marrow trephine


biopsy.

Constituents of bone marrow : RBC, WBC, PLT and CT (collagen, fibrous tissue)

What can happen in to marrow : aplasia


Hyperplasia
SOL - primary, metastatic

Aplastic Anaemia

Aetiology– idiopathic, drugs, chemicals, irradiation and viruses


Pathology of bone marrow/metabolic disease 2013
Page 2

Bone Marrow: aplastic


anemia

Hyperplastic marrow
B12 and iron deficiency.

Space occupying lesions:


Leukaemia
types – acute and chronic
acute – ALL, AML
chronic – CLL, CML

Myeloproliferative disorders

Normal control mechanisms for governing the cell lines involved are no longer active, mutated
tyrosine kinases lead to growth factor independent proliferation. This does not impair
differentiation and causes clonal proliferation of one or more haemopoietic cell lineages.

Common features of myeloproliferative disorders:


1. Increased proliferative drive in bone marrow
2. Extramedullary haematopoiesis
3. Variable transformation to myelofibrosis
4. Variable transformation to acute leukaemia

Specific examples:
PRV – RBC overproduction
Essential thrombocytosis - increased megakaryocytes
Primary myelofibrosis – overproduction of collagen or fibrous tissue
CML – overproduction of granulocytes, Philadelphia chromosome

Leukemia
Pathology of bone marrow/metabolic disease 2013
Page 3

Myelodysplastic disorders

Group of disorders characterized by clonal stem cell abnormality, leading to maturation defects
associated with ineffective haematopoiesis. This results in morphological abnormalities in blood
cells and an increased risk of transformation to acute myelogenous leukaemia.
Hallmark is cellular bone marrow with clonal progeny of mutant stem cell, which has the ability
to differentiate into RBC, WBC and platelets but in an ineffective and disordered fashion
(cytopenias in peripheral blood). Usually occurs in elderly and main complications are anaemia
and infection.

Multiple myeloma

Most common plama cell neoplasm.


Disease of elderly 65 – 70 years old.
Secrete monoclonal immunoglobulin.
BJ proteins in urine
Multifocal destructive bone lesions esp. axial skeleton.
X-ray – osteolytic lesions
Complications – hypercalcaemia (TNF increases osteoclastic activity), amyloidosis, pathological
fracture, infection, plasma cell leukaemia, myeloma kidney

(Left)Spine from a man aged 62


with multiple myeloma. Bone
marrow aspirate (Right) showed
marrow replacement by plasma
cells
Pathology of bone marrow/metabolic disease 2013
Page 4

Skull from a woman


aged 62

Metabolic bone diseases

Affects entire skeleton.

Main conditions hyperparathyroidism, rickets, osteomalacia and osteoporosis.

Hyperparathyroidism - primary, secondary and tertiary


- skeletal changes
- osteitis fibrosa cystica

Vit D
Maintains plasma Ca and Phosphorus levels.
Decrease - Ricketts (children)
- osteomalacia (adults)
Source of Vit D

Pathogenesis

Osteoporosis

Increased porosity of the skeleton, due to decrease in bone mass.

Primary - senile, post-menopausal, idiopathic


Secondary - endocrine – hyperPTH
- neoplasia – multiple myeloma
- malnutrition
- drugs – steroids
Pathology of bone marrow/metabolic disease 2013
Page 5

Renal osteodystrophy
Term used to describe all skeletal changes of chronic renal failure.

phosphate
retention

 calcium
CRF  PTH (secondary
Hyperparathyroidism)

↓active vitamin D ↓calcium


Pathology of bone marrow/metabolic disease 2013
Page 6

PATHOLOGY OF BONE MARROW/METABOLIC BONE DISEASE


Demonstration Pots
Royal Brisbane Clinical School

Dissection of the neck (patient died of chronic renal


failure)
_____________________________________________
_____________________________________________
_____________________________________________
_____________________________________________
_____________________________________________
_____________________________________________

Parathyroid mass
_______________________________
_______________________________
_______________________________
_______________________________
_______________________________
_______________________________

Femur from a patient with parathyroid adenoma.


_________________________________________________
_________________________________________________
_________________________________________________
_________________________________________________
_________________________________________________
_________________________________________________
_________________________________________________
Pathology of bone marrow/metabolic disease 2013
Page 7

Lesion in the spine


___________________________
___________________________
___________________________
___________________________
___________________________
___________________________
___________________________
___________________________

58 yrs /M with anaemia and raised


platelet count for 6 years before death.
The spleen was very large, and
histologically showed extensive
extramedullary haemopoiesis and
numerous megakaryocytes.
_________________________________
_________________________________
_________________________________
_________________________________
_________________________________
_________________________________
_________________________________

Rib lesion 41yrs/F with a parathyroid adenoma. Serum


calcium late in her illness was 3.87 mmol/l.
_____________________________________________
_____________________________________________
_____________________________________________
_____________________________________________
_____________________________________________
_____________________________________________
_____________________________________________
_____________________________________________
_____________________________________________
Pathology of bone marrow/metabolic disease 2013
Page 8

Spine 47yrs/M
________________________________
________________________________
________________________________
________________________________
________________________________
________________________________
________________________________
________________________________
________________________________

Spleen from 77 yrs/M. It weighed 850g. The


results of haematological investigations performed
just before death were as follows: blood
haemoglobin concentration, 4g/dl; white cell
9
count, 20x10 /l of which 70% were myeloblasts;
9
platelet count, 5x10 /l.
________________________________________
________________________________________
________________________________________
________________________________________
________________________________________
________________________________________
________________________________________
________________________________________
________________________________________
________________________________________
________________________________________
Pathology of bone marrow/metabolic disease 2013
Page 9

PATHOLOGY OF BONE MARROW/METABOLIC BONE DISEASE


Demonstration Pots
PA/Mater Clinical School

femoral marrow in a woman with iron deficiency


anaemia.
________________________________________
________________________________________
________________________________________
________________________________________
________________________________________
________________________________________

Bones from a 58 yrs/M. The patient


suffered from anaemia for 6 years before
death. Blood platelet counts ranged up to
1,000 x 109/l, and large megakaryocyte
fragments were present in the
circulation. The spleen was very large,
and contained numerous myeloid and
erythroid precursors and
megakaryocytes.
________________________________
________________________________
________________________________
________________________________
________________________________
________________________________
________________________________
________________________________
________________________________
Pathology of bone marrow/metabolic disease 2013
Page 10

Skull 49 yrs/M
_______________________________________
_______________________________________
_______________________________________
_______________________________________
_______________________________________
_______________________________________
_______________________________________

Spine 62 yrs/F 15 month history of bone pain.


_____________________________________
_____________________________________
_____________________________________
_____________________________________
_____________________________________
_____________________________________
_____________________________________
_____________________________________

Femur:
____________________________
____________________________
____________________________
____________________________
____________________________
____________________________
Pathology of bone marrow/metabolic disease 2013
Page 11

Spleen 6 Kgs:
________________________________________
________________________________________
________________________________________
________________________________________
________________________________________
________________________________________
________________________________________

Spine 47 yrs/M
_______________________________________
_______________________________________
_______________________________________
_______________________________________
_______________________________________
_______________________________________
_______________________________________
_______________________________________
Pathology of bone marrow/metabolic disease 2013
Page 12

PATHOLOGY OF BONE MARROW/METABOLIC BONE DISEASE


Demonstration Pots
Ipswich Clinical School

Skull:
______________________________________________
______________________________________________
______________________________________________
______________________________________________

Spleen:
___________________________________________
___________________________________________
___________________________________________
X316E.
X316E. Spleen:
Spleen: chronic
chronic
___________________________________________
myelocytic
myelocytic leukaemia
leukaemia
___________________________________________
___________________________________________
___________________________________________

Spine 70 yrs/F
______________________________________
______________________________________
______________________________________
______________________________________
______________________________________
______________________________________
______________________________________
______________________________________
______________________________________
______________________________________
______________________________________
Pathology of bone marrow/metabolic disease 2013
Page 13

Parathyroid removed from patient


with chronic renal failure
___________________________
___________________________
___________________________
___________________________
___________________________
___________________________

Femur 67yrs/F died due to intracerebral


haemorrhage.
________________________________
________________________________
________________________________
________________________________
________________________________
________________________________

Femur 49yrs/M
____________________________________
____________________________________
____________________________________
____________________________________
Pathology of skin tumours 2013
Page 1

MBBS2 PATHOLOGY TUTORIAL


PATHOLOGY OF SKIN TUMOURS

Learning objectives:

Tumours of melanocyte, tumours of keratinocytes


Major macroscopic and microscopic features of skin neoplasia
Understand the role of skin biopsy
Tumours of skin divided into two groups, tumours from melanocytes and tumours from
keratinocytes

General

Skin pathology encompasses a wide variety of pathological conditions.


Other important factors when assessing skin lesions:

Two major layers of skin : epidermis and dermis – derived from different embryonic
components therefore morphologically different.
Epidermis is stratified, cellular , avascular, lacks nerves and sits on a basement
membrane. Adnexae eg. Hair follicles extend from epidermis into the dermis.

Histology of Skin

Epidermis – 5 layers, sweat glands, hair follicles, sebaceous glands

Dermis is divided into papillary and reticular dermis.

Subcutaneous fat.

Microscopic definitions:
Hyperkeratosis
Parakeratosis
Hypergranulosis
Acanthosis
Acantholysis
Spongiosis
Exocytosis
Pathology of skin tumours 2013
Page 2

The clinical sign is related to the underlying pathology.


Eg. Scaling – hyperkeratosis, parakeratosis
Blister – spongiosis

Pigmented skin lesions

Melanocytes
– migrate from neural crest to dermo-epidermal junction
Lesions without proliferation of melanocytes – freckles, melasma
Lesions with proliferation of melanocytes – Lentigo, Melanocytic naevi, Dysplastic naevi

Intradermal naevus.

Dysplastic naevi.

Malignant melanoma
Pathogenesis
Clinocopathological features
Clinical changes
Tumour progression
Main 4 types
Histopathology – Breslow thickness, Clarke’s levels, completeness of excision
Clinical course – metastases, sentinel node biopsy
Pathology of skin tumours 2013
Page 3

Epithelial Tumours
Skin tag (fibroepithelial polyp)
Seborrhoeic keratosis
Keratoacanthoma
Solar keratosis – pathogenesis
Intraepidermal carcinoma
BCC
SCC

Solar keratosis. A solar keratosis


excised from the face.

Carcinoma in situ.

BCC
Pathology of skin tumours 2013
Page 4

SCC.

BCC
Pathology of skin tumours 2013
Page 5

Melanoma

*The following conditions will be discussed if there is time*

Inflammatory Dermatoses

Acute and Chronic

Acute – lasts days to weeks, characterized by inflammation, oedema and epidermal,


vascular or subcutaneous injury. Examples: urticaria, acute eczematous dermatitis,
erythema multiforme,
Chronic – last months to years, often show altered epidermal growth eg hyperplasia.
Examples: psoriasis, seborrhoeic dermatitis, lichen planus.

Bullous disorders:

Blistering dermatoses are characterized by fluid filled spaces with the skin. The level at
which the vesicle or bullous occur is important for histological diagnosis.

Examples:

Subcorneal: impetigo
Pathology of skin tumours 2013
Page 6

Reproduced from Robbins &


Cotran Pathologic Basis of
Disease 7th Ed. 2005

Suprabasal: pemphigus vulgaris

Reproduced from Robbins &


Cotran Pathologic Basis of
Disease 7th Ed. 2005

Subepidermal: bullous pemphigoid

Reproduced from Robbins &


Cotran Pathologic Basis of
Disease 7th Ed. 2005

Clinically the more superficial the vesicle or bullous the more likely it is to rupture and
hence the lesions will be “weepy”.
Pathology of skin tumours 2013
Page 7

PATHOLOGY OF SKIN TUMOURS


Demonstration Pots
Royal Brisbane Clinical School

spine 42yrs/M.
____________________________________
____________________________________
____________________________________
___________________________________

92yrs/F Skin fore-arm


_______________________________
_______________________________
_______________________________
_______________________________
_______________________________
Pathology of skin tumours 2013
Page 8

42 yrs/M scaly lesions face


_____________________________
_____________________________
_____________________________
_____________________________
_____________________________
_____________________________
_____________________________

66 yrs/M
________________________________
________________________________
________________________________
________________________________

51 yrs /M
______________________________
______________________________
Pathology of skin tumours 2013
Page 9

PATHOLOGY OF SKIN TUMOURS


Demonstration Pots
PA/Mater Clinical School

76 yrs/F: Pigmented lesion from back with


itchiness
__________________________________
__________________________________
__________________________________
__________________________________
__________________________________
__________________________________
__________________________________

54 yrs/F pigmented lesion leg


_______________________________________
_______________________________________
_______________________________________
_______________________________________
_______________________________________
Pathology of skin tumours 2013
Page 10

Amputated toe
________________________________
________________________________
________________________________
________________________________
________________________________
________________________________

Amputated hand
_______________________________________
_______________________________________
_______________________________________
_______________________________________
_______________________________________

68 yrs/M lesion from the site of a chronic


varicose ulcer, which has been present on
the leg for 15 years.
___________________________________
___________________________________
____
Pathology of skin tumours 2013
Page 11

PATHOLOGY OF SKIN TUMOURS


Demonstration Pots
Ipswich Clinical School

Amputated specimen hand


________________________________
________________________________
________________________________

Spine 29 yrs /M
_________________________________
_________________________________
_________________________________
_________________________________
_________________________________
_________________________________

Ulcerated haemangioma of the foot in


a man aged 64. Operative surgical
specimen.
Pathology of skin tumours 2013
Page 12

Amputated hand
_________________________________________
_________________________________________
_________________________________________
_________________________________________

Multiple pigmented coin like lesions with “stuck


on” appearance
_________________________________________
_________________________________________
_________________________________________

57/F lower leg.


____________________________
____________________________
____________________________
____________________________
____________________________
Pathology of skin tumours 2013
Page 13

Skin eyelids. Tumour invading


eylids and orbit.
___________________________
___________________________
___________________________
__________________________
Cardiovascular I: Ischemic and hypertensive heart disease 2013
Page 1

MBBS2 Tutorial – Cardiovascular I Tutorial

Ischemic and Hypertensive Heart Disease

Learning objectives

Pathogenesis of ischemia
Morphological changes and complications of infarction
Pathogenesis and complications of hypertensive heart disease

Anatomy and blood supply of the heart

Definition of Ischemia – imbalance between supply (perfusion) and demand for oxygenated
blood.
This may come about by impaired blood supply (90% of cases of ischemia are due to
atherosclerotic coronary arterial obstruction) or the myocardium becomes hypertrophic
resulting in a greater demand on the blood supply.
The myocardium may hypertrophy as an adaptive response eg. in systemic hypertension or
idiopathic e.g. cardiomyopathy.
Myocardium is very active and 30% of the muscle fibres are made up of mitochondria
therefore aerobic metabolism is essential.

Compensatory mechanisms;
1. Autoregulatory mechanism controls blood flow through the coronary vascular bed.
When obstruction occurs in a major coronary vessel, the peripheral resistance distal
to the obstruction is reduced so that blood flow is maintained.
2. If obstruction occurs over a period of time collateral circulation develops.

In order for ischemia to develop at least 75% of the lumen must be obstructed.

Pathogenesis of atherosclerosis
Cardiovascular I: Ischemic and hypertensive heart disease 2013
Page 2

(Reproduced from Robbins & Cotran Pathologic Basis of Disease 7th Ed. 2005)
Sequential progression of lesion in coronary artery causing the four clinical manifestations
of IHD

(Reproduced from Robbins & Cotran Pathologic Basis of Disease 7th Ed. 2005)

Clinical manisfestation of IHD, four syndromes:

1. Angina pectoris
2. Myocardial infarction – types (transmural, subendocardial)
- role of plaque change
- light microscope changes

4 – 12 hours

1 – 3 days
Cardiovascular I: Ischemic and hypertensive heart disease 2013
Page 3

4 – 9 days: Macrophages being phagocytosis, development of granulation tissue.

10 days onwards:

Weeks to months

- consequences and complications


3. Chronic ischaemic heart disease
4. Sudden cardiac death.

Hypertensive Heart Disease


Definition and pathogenesis (adaptive response to pressure overload)
Criteria for diagnosis

Specimen from 62 yrs/M

Morphology of hypertensive heart disease


Complications
Cardiovascular I: Ischemic and hypertensive heart disease 2013
Page 4

CVI – PATHOLOGY OF ISCHAEMIC HEART DISEASE & HYPERTENSION


Demonstration Pots
Royal Brisbane Clinical School

Heart of a 20 year old woman


who was killed in a motor
vehicle accident.

A 60 year old woman experienced the


sudden onset of severe pain in the chest
and left arm.

Sudden onset of severe chest pain


since two weeks in a hypertensive
woman aged 42.

Chest pain in a man aged 58 with a history of


hypertension and of having smoked 20 cigarettes a day.
Cardiovascular I: Ischemic and hypertensive heart disease 2013
Page 5

A 58 year old man


with severe chest pain
and dyspnea.

60 yrs male with old


H/O IHD

Man aged 72 with a past history


of MI underwent CABG
(coronary artery bypass graft)
and died after 3 weeks
Cardiovascular I: Ischemic and hypertensive heart disease 2013
Page 6

Patient with acute myocardial


infarction (AMI) developing
pansystolic murmur 4 days later
1
2

Cardiovascular I: Ischemic and hypertensive heart disease 2013


Page 7

CVI – PATHOLOGY OF ISCHAEMIC HEART DISEASE & HYPERTENSION


Demonstration Pots
PA/Mater Clinical School

70 year old man with h/o severe


breathlessness.

Autopsy specimen 5
days after chest pain.

Patient with acute myocardial infarction


(AMI) developed pansystolic murmur 5
days later.
1
2

Cardiovascular I: Ischemic and hypertensive heart disease 2013


Page 8

Autopsy specimen from a 68 year


old hypertensive and diabetic man.

A 65 year old man with a


long H/O hypertension.

A 67 year old man with prolonged


H/O hypertension and severe
dyspnea.
Cardiovascular I: Ischemic and hypertensive heart disease 2013
Page 9

CVI – PATHOLOGY OF ISCHAEMIC HEART DISEASE & HYPERTENSION


Demonstration Pots
Ipswich Clinical School

A patient who died 5 days after onset of


severe chest pain.

A 70 year old man with H/O


IHD, hypertension and dyspnea

An autopsy specimen of a patient


following trauma to the anterior chest
wall.
Cardiovascular I: Ischemic and hypertensive heart disease 2013
Page 10

A 55 year old man with H/O severe


hypertension.

A 78 year old man with chest pain.

Woman aged 77 was found to have


blood stained fluid in the pericardial
cavity at autopsy.
Cardiovascular II: Congestive cardiac failure, pulmonary hypertension, cor pulmonale, pericardial disease, and tumours 2013
Page No. 1

MBBS2 Tutorial – Cardiovascular System II


Congestive cardiac failure, pulmonary hypertension, cor pulmonale, pericardial disease,
and tumours.

Learning Objectives:

To understand the pathogenesis of congestive cardiac failure and cor pulmonale.


Know and understand other cardiac disorders including cardiomyopathy, myocarditis,
pericardial disease and tumours.

CCF

Occurs when CO does not equal demand. Common endpoint for many myocardial,
valvular, congenital and some extracardiac diseases and results in hypertrophy/dilatation.

You need to consider why the heart has failed:


1. failure of pump - MI
2. obstruction to flow – AS
3. regurgitant flow – AR
4. disorders of conduction – arrhythmia
5. disruption of circulatory system – gunshot wound, # femur – blood loss

Compensation:
1. Frank-Starling : increased preload dilation leads to increased contraction
2. hypertrophy
3. increased heart rate via neurotransmitters eg adrenaline

When exceeded = CCF

(Reproduced from Robbins & Cotran Pathologic Basis of Disease 7th Ed. 2005)
Cardiovascular II: Congestive cardiac failure, pulmonary hypertension, cor pulmonale, pericardial disease, and tumours 2013
Page No. 2

X1194N. 76 year old man with polyostotic Paget's


disease of bone. Blood pressure normal. No evidence
of ischaemic heart disease.

Clinicopathological features – left sided heart failure - pathogenesis and relevant


morphological changes

1. dyspnoea and lung pathology


2. kidney – fluid retention, renin angiotensin aldosterone system
3. brain – hypoxia in severe failure

(right sided heart failure considered under cor pulmonale)

Pulmonary hypertension

Pulmonary circulation – low resistance, one eighth systemic pressure


Pulmonary hypertension = when pulmonary pressure reaches one quarter

May be Primary – cause unknown


Secondary – most frequent
- increased blood flow or pressure – mitral stenosis
- increased pulm vascular resistance – emboli
- left heart resistance to blood flow – systemic sclerosis

Cor pulmonale

RV hypertrophy and/or dilatation due to pulmonary hypertension (due to lung or


pulmonary vasculature disease)
Definition excludes dilatation caused by congenital heart disease or by diseases of left
side of heart.

May be acute or chronic.


Depends on suddenness : acute – massive pulm embolus (dilatation)
chronic – due to prolonged pressure overload
Morphology differs, in chronic have hypertrophy, acute only dilation.
Cardiovascular II: Congestive cardiac failure, pulmonary hypertension, cor pulmonale, pericardial disease, and tumours 2013
Page No. 3

Predisposing disorders :
1. lung parenchyma – cystic fibrosis, diffuse interstitial pulm fibrosis
2. pulmonary vasculature – extensive thromboembolism
3. disorders affecting chest wall movement – kyphoscoliosis
4. pulmonary arterial constriction – hypoxia, metabolic acidosis

Clinicopathological features – right sided heart failure - pathogenesis and relevant


morphological changes
1. hepatomegaly – passive congestion
2. splenomegaly – elevated pressure in portal vein
3. peripheral oedema
4. pleural effusions

Myocardial disease

Cardiomyopathy
Brief description of types, include pathogenesis and morphology.
Use of endomyocardial biopsies.

Myocarditis
Definition
Major causes – elaborate on common ones

lymphocytic infiltration hypersensitivity myocarditis,


inflammatory infiltrate mainly eosinophils

giant cell myocarditis

(Reproduced from Robbins & Cotran Pathologic Basis of Disease 7th Ed. 2005)
Cardiovascular II: Congestive cardiac failure, pulmonary hypertension, cor pulmonale, pericardial disease, and tumours 2013
Page No. 4

Pericardial disease

Pericardial effusion and haemopericardium


Pericarditis – acute vs chronic
- causes
- morphology

Cardiac tumours

Primary and Metastatic


Most common primary tumours are benign. Important ones include-
- Myxoma
- Rhabdomyoma
-
Effects of non-cardiac neoplasms on the CVS eg SVC syndrome
Cardiovascular II: Congestive cardiac failure, pulmonary hypertension, cor pulmonale, pericardial disease, and tumours 2013
Page No. 5

CVII – CONGESTIVE CARDIAC FAILURE, PULMONARY HYPERTENSION,


COR PULMONALE, PERICARDIAL DISEASE AND TUMOURS
Demonstration Pots
Royal Brisbane Clinical School

Man aged 59 years.

Female aged 17 years. The patient


collapsed at a disco, and died 2 hours
later.

This patient presented with signs of


heart failure 6 months before death.
Cardiovascular II: Congestive cardiac failure, pulmonary hypertension, cor pulmonale, pericardial disease, and tumours 2013
Page No. 6

X265B.

Woman aged 72 with long-standing heart


failure. Death was due to coincidental
perforation of a chronic duodenal ulcer.

Man aged 64. He had smoked 60 cigarettes a day


for 40 years, but had given them up in the terminal
stages of his illness. At the time of his final
admission to hospital, the pH of his arterial blood
was 7.16.
Cardiovascular II: Congestive cardiac failure, pulmonary hypertension, cor pulmonale, pericardial disease, and tumours 2013
Page No. 7

Previously healthy man aged 33 who died


suddenly. The heart weighed 550 g
Cardiovascular II: Congestive cardiac failure, pulmonary hypertension, cor pulmonale, pericardial disease, and tumours 2013
Page No. 8

CVII – CONGESTIVE CARDIAC FAILURE, PULMONARY HYPERTENSION,


COR PULMONALE, PERICARDIAL DISEASE, AND TUMOURS
Demonstration Pots
PA/Mater Clinical School

Patient had a history of chronic renal


failure.

76 year old man with polyostotic Paget's


disease of bone. Blood pressure normal.
No evidence of ischaemic heart disease.
Cardiovascular II: Congestive cardiac failure, pulmonary hypertension, cor pulmonale, pericardial disease, and tumours 2013
Page No. 9

Patient had a history of atrial fibrillation.

What might be the clinical effects of this


lesion?

Young woman. She died soon


after admission to hospital in a
profoundly shocked state.
Cardiovascular II: Congestive cardiac failure, pulmonary hypertension, cor pulmonale, pericardial disease, and tumours 2013
Page No. 10

What am I? Clinical features of


congestive cardiac failure may be
found in this condition.
Cardiovascular II: Congestive cardiac failure, pulmonary hypertension, cor pulmonale, pericardial disease, and tumours 2013
Page No. 11

CVII – CONGESTIVE CARDIAC FAILURE, PULMONARY HYPERTENSION,


COR PULMONALE, PERICARDIAL DISEASE, AND TUMOURS
Demonstration Pots
Ipswich Clinical School

The myocardial discoloration is an


artefact.

An incidental finding in a
woman aged 40 killed in a
motor vehicle accident.
Cardiovascular II: Congestive cardiac failure, pulmonary hypertension, cor pulmonale, pericardial disease, and tumours 2013
Page No. 12

Man aged 53. Smoker.

Woman aged 61 with chronic renal


failure and long-standing
hypertension. Two days before
death her blood urea was 206mg.%.

60 year old female patient


2013 - CVSIII – Thrombosis & Embolism; Pathology of arteries & veins
Page 1
MBBS2 Tutorial – Cardiovascular III
Thrombosis and Embolism; Pathology of arteries and veins

Learning Objectives:
Know and understand predisposing factors to thrombosis and embolism
Outcome and complications of thrombi and emboli
Pathology of aneurysms
Classification of vasculitis

Template

Anatomy of blood vessels

Artery thicker than vein as need to withstand a higher pressure.


As arteries become narrower the wall thickness diminishes.

Wall – endothelial cells


- smooth muscle
- extracellular matrix – collagen, elastin etc..
3 concentric layers - intima, media, adventitia
Intima – endothelial cells, minimal subendothelial CT
- internal elastic lamina separates it from the media
Media – smooth muscle
- vasa vasorum (vessels of vessels)
- external elastic lamina separates it from the adventitial
Adventitia – CT, nerves, vasa vasorum

Main Pathology of vessels:

Thrombosis
Embolism
Arteriosclerosis
Anuerysm
Vasculitis

Thrombosis
Definition of thrombus
Pathogenesis of thrombi – Virchow triad
2013 - CVSIII – Thrombosis & Embolism; Pathology of arteries & veins
Page 2

Morphology of thrombi

Arterial (white) thrombus (arrow)


comprising mainly platelets and
fibrin.

Outcome and complications of thrombi

(Left). Occlusive venous thrombus. Note


coralline thrombus with alternating
layers of platelets and fibrin (arrowhead)
and consecutive clot (arrow) that forms
after vessel occlusion.
(Right). Organizing thrombus. Note
ingrowth of vessels and fibroblasts (blue
arrows) into lumenal thrombus (black
arrows). Some collagen deposition has
occurred.
2013 - CVSIII – Thrombosis & Embolism; Pathology of arteries & veins
Page 3

Organized thrombus (arrows) in


femoral vein. Note how the
fibrous bands interfere with the
functional integrity of valves,
thus producing deep venous
insufficiency.

Embolus
Definition of embolus
Types of emboli

X3015C. Brain in fatal fat


embolism syndrome.

(Left). Symetrically round, (empty)


pulmonary vessels (arrows) in fat
embolism.
(Right). Oil Red O stain on frozen
lung tissue demonstrating fat
(arrows) in pulmonary vessels.
1
2

2013 - CVSIII – Thrombosis & Embolism; Pathology of arteries & veins


Page 4

Massive pulmonary
embolism.

Outcomes and complications

Ateriosclerosis:
Arterial wall thickening and loss of elasticity (literally means hardening of arteries.
3 patterns
1. Atherosclerosis
2. Monckeberg medial calcific sclerosis
3. Arteriolosclerosis (assoc primarily with hypertension)

Pathogenesis of Atherosclerosis
Risk factors

Lipid streaks (arrows) in the


aorta of a male aged 21 years.
These are the earliest
manifestation of
atherosclerosis.
2013 - CVSIII – Thrombosis & Embolism; Pathology of arteries & veins
Page 5

Lipid (fatty) streak. Note


subendothelial collection of
'foam' cells (F). The cells appear
to be empty because the lipid has
been dissolved out in the
processing of the tissue.

(Left) Aorta: atheroma. Fibro-fatty


plaques (arrows).The next stage in
the development of atherosclerosis.
(Right) Fibrofatty plaque. Note
fibrous cap (black arrow) and
underlying lipid in form of
cholesterol crystals (blue arrows).

(Left). Close-up of fibrofatty plaque.


2013 - CVSIII – Thrombosis & Embolism; Pathology of arteries & veins
Page 6

Male aged 74 Complicated atheroma


of aorta

(Left) Atheromatous aorta with thrombosis.


Man aged 64.
(Right) Atheroma of the arch of the aorta and
branches with thrombosis of the right internal
carotid artery in a man aged 69. Autopsy
disclosed old and recent infarction of the right
cerebral hemisphere. This is one cause of
cerebral infarction (stroke)

Marked atheromatous stenosis of


orifice of right renal artery (blue
arrow) with advanced ischaemic
atrophy of right kidney. B.P. 180/105.
Blood urea 8.6 mmol/l. Male aged 84.
Renal artery stenosis is one cause of
hypertension.

Complications of atherosclerosis

Monckeberg medial calcific sclerosis

Arterioslcerosis
Role of hypertension – benign vs malignant

Anuerysms
Definition
True vs false
2013 - CVSIII – Thrombosis & Embolism; Pathology of arteries & veins
Page 7

(Left) Complicated atheroma with aneurysm of


the lower abdominal aorta. Woman aged 81 with
chronic renal failure and hypertension. Note
thrombus (blue arrow) within aneurysm (red
arrow).
(Right) Atheromatous aneurysm of the abdominal
aorta with thrombosis. Note lines of Zahn on its
surface (arrows)
Sudden rupture of an abdominal aneurysm causes
death, but the rupture may occur slowly as a
continuous leak that presents as abdominal pain.
Abdominal aneurysms can be repaired surgically
and this is a common operative procedure.

(Left) Aortic dissection in a hypertensive male of


71. A large amount of blood clot was found in the
pericardial sac at autopsy. Note the myocardial
hypertrophy and the aortic intimal tear (arrow).
(Right) Aortic dissection: note compression of
lumen of innominate artery (arrow), best seen from
the top of the pot. Hypertensive left ventricular
hypertrophy.

Outcomes of aneurysm

Vasculitis
Classification according to size of vessel.

(Left) Polyarteritis nodosa. Male aged 12 years.


Note the beaded appearance (arrows) of the
coronary arteries. The inflamed wall bulges
forming an aneurysm seen as nodules
(Right) Polyarteritis nodosa. There is segmental
necrosis (arrow) of the walls of small and medium
sized muscular arteries and perivascular
inflammation.
2013 - CVSIII – Thrombosis & Embolism; Pathology of arteries & veins
Page 8
CARDIOVASCULAR III
Demonstration pots
Royal Brisbane Clinical School

A man aged 93 with gangrene of


the left hallux and ulceration of the
left lower leg.

A 70 year old man had severe


abdominal pain and was in
shock on arrival to
emergency.

A man aged 86 with dementia,


cardiac failure and lower limb
venous thrombosis.
1
2

2013 - CVSIII – Thrombosis & Embolism; Pathology of arteries & veins


Page 9

A man aged 62 treated successfully


with penicillin for acute bacterial
endocarditis of the aortic valve 5
months before death.

A 60 year old man with


syphilis was in hypovolemic
shock during admission.

Aorta of a 69 year old


hypertensive & diabetic
man.
2013 - CVSIII – Thrombosis & Embolism; Pathology of arteries & veins
Page 10

Aorta of a 75 year old woman


who died of acute myocardial
infarction.

A man with long standing diabetes


mellitus complained of pain in lower
limbs.
1
2

2013 - CVSIII – Thrombosis & Embolism; Pathology of arteries & veins


Page 11
CARDIOVASCULAR III
Demonstration Pots
PA/Mater Clinical School

Patient had metastatic carcinoma


with pancreatic deposits.

The patient suffered from


ulceration of the legs and had large
collateral veins of abdominal wall.

Patient with high cholesterol


levels died of acute myocardial
infarction.
2013 - CVSIII – Thrombosis & Embolism; Pathology of arteries & veins
Page 12

A 68 year chronic smoker with


signs & symptoms of systemic
thromboembolism.

A 78 year old hypertensive man


presented with abdominal pain
& shock.

A 72 year old man


presented with feeble pulses
and BP of 70/40.

A woman aged 66 who died 10 days after


developing a myocardial infarct.
2013 - CVSIII – Thrombosis & Embolism; Pathology of arteries & veins
Page 13

A man aged 56 had


myocardial infarction, heart
failure and left atrial
thrombosis.

A 65 year old man with syphilis.


2013 - CVSIII – Thrombosis & Embolism; Pathology of arteries & veins
Page 14
CARDIOVASCULAR III
Demonstration Pots
Ipswich Clinical School

Thoracic and abdominal aorta in


an old hypertensive man.

A 78 year old diabetic woman with


atrial fibrillation and left atrial
thrombosis.

A 65 year old hypertensive male


experienced the sudden onset of severe
central chest pain that radiated to the
back and legs four days before death.

Man aged 84 who died suddenly one


month after repair of a ruptured
abdominal aortic aneurysm.
2013 - CVSIII – Thrombosis & Embolism; Pathology of arteries & veins
Page 15

A man aged 50 with carcinoma of the


pancreas with swollen leg.

A 50 year old woman had


severe chest pain and died
after a short while.
2013 - Cardiovascular IV – Valvular heart disease, infective endocarditis & congenital heart disease
Page 1
MBBS 2 Tutorial – Cardiovascular IV
Valvular heart disease, infective endocarditis & congenital heart disease

Learning Objectives :
Understand types of valvular defects
Pathogenesis and outcome of rheumatic heart disease and infectious endocarditis
Know pathology of congenital heart disease

Review normal valve anatomy and function

Valvular heart diseases

Types :

Regurgitation – failure of valve to close causing reversed blood flow


Stenosis – failure of valve to open completely, impeding forward flow

Pure or mixed disease


Isolated (one valve) or combined (more than one)

Stenosis is almost always due to a primary valve disease, whereas regurgitation may be due to valve
disease or fuctional.
Functional regurgitation – valve incompetent due to 1. dilation of the ventricle or 2. dilation of aortic or
pulmonary artery

Clinical consequence depends on – degree of abnormality


- rate of development
- rate and quality of compensatory mechanism

Discuss:
Mitral incompetence – floppy mitral valve

Floppy mitral valve (arrows) in a man


aged 56 who died suddenly and
unexpectedly. There was a past history
of syncopal attacks. Microscopy
showed an increase in myxoid ground
substance in the valve leaflets.

Mitral stenosis
Aortic incompetence
Aortic stenosis esp. calcific aortic valve disease, bicuspid aortic valve
2013 - Cardiovascular IV – Valvular heart disease, infective endocarditis & congenital heart disease
Page 2

Stenosed, calcified, bicuspid aortic valve. Woman


aged 66. Heart weighed 610 g.
This condition can be found incidentally without
any history of rheumatic fever.

Approach to Valve vegetations

1. RHD – small, warty

2. IE – large, bulky, irregular

3. NBTE – bland

4. Libman Sacks – small to medium, both sides of valve

(Reproduced from Robbins & Cotran Pathologic Basis of Disease 7th Ed. 2005)

Rheumatic Heart Disease


Pathogenesis
Clinical diagnosis
2013 - Cardiovascular IV – Valvular heart disease, infective endocarditis & congenital heart disease
Page 3

Active rheumatic carditis in a girl aged 11. There was a 4


year history of recurrent attacks of rheumatic fever and of
persistent mitral and aortic incompetence with severe heart
failure. Death occurred suddenly. Note the thickening of the
mitral (blue arrows) and aortic cusps (black arrows) with
rows of vegetations near their edges, the fibrosis of the left
atrial endocardium, the dilatation and hypertrophy of all
cardiac chambers, and the chronic obliterative pericarditis.
Tiny vegetations were also present on the atrial surface of the
tricuspid valve.

Discuss histology

Histology - Rheumatic myocarditis. The


classical Aschoff body is seen here. The cells
are located in the perivascular region within the
myocardium. The ‘Anitschkow’ cells are plump
activated macrophages (arrowed in B), and these
are admixed with other inflammatory cells. The
‘Anitchkow’ cells have chromatin arranged in a
central ribbon and this has been likened to the
appearance of a caterpillar (hence "caterpillar
cells"). [Aschoff was from Freiburg and
Anitchkow was from St. Petersburg.]

Discuss complications and outcomes

Chronic rheumatic heart disease with involvement


of aortic, mitral (arrows) and tricuspid valves in a
woman of 61. She had had four attacks of acute
rheumatic fever at the ages of 4, 14, 18 and 30
years. Exertional dyspnoea was first noted when
she was 30: Nocturnal dyspnoea and ankle
swelling began at the age of 50. She died of heart
failure at 61.
The first symptoms of the presence of rheumatic
heart disease are often seen during pregnancy.
2013 - Cardiovascular IV – Valvular heart disease, infective endocarditis & congenital heart disease
Page 4
Infectious Endocarditis
Pathogenesis

Infective endocarditis due to Streptococcus


viridans in a woman aged 35. Note the
multiple vegetations (arrows). S. viridans is
probably the commonest organism to cause
infective endocarditis. It has a low degree of
virulence. Formerly this form of endocarditis
was called sub acute bacterial endocarditis.

A.B. Infective endocarditis. Features to


note are the thickened fibrotic valve
leaflet (black arrow), the vegetation
(blue arrows) containing pink fibrin and
platelets, and the dark blue colonies of
bacteria (red arrow), better seen in the
higher power photomicrograph in B.

A B

Discuss complications
2013 - Cardiovascular IV – Valvular heart disease, infective endocarditis & congenital heart disease
Page 5

Perforation (arrow) of mitral


cusp due to Staphylococcus
aureus endocarditis.

Other valvular heart disease – SLE, marantic vegetations

Valve Prosthesis
Types
Complications
Mitral and aortic valve prostheses (arrow) inserted in a
man aged 49 with heart failure due to chronic valve
disease following rheumatic fever at the age of 3 years.
During the 1960’s definitive surgery for damaged heart
valves was introduced. Many different types of valve
were tested. Some of them caused immunological
rejection but ultimately the valves were made of inert
material. Calf valves were used with good lasting
qualities in some centres.
The spouses of some patients were worried by the noise

Congenital heart disease


Refers to heart and great vessels

Main problems are either shunts or obstruction to blood flow


Shunts may be right to left or left to right
R to L:
Discuss why cyanosis occurs
Explain paradoxical embolism
Fallot’s tetralogy
Transpostition of great vessels

L to R:
ASD
VSD
PDA
2013 - Cardiovascular IV – Valvular heart disease, infective endocarditis & congenital heart disease
Page 6
AV septal defect

Patent ductus arteriosus. Marked right


ventricular hypertrophy. Atheroma of pulmonary
artery due to severe pulmonary hypertension.
This is an example of Eisenmenger's syndrome
complicating a shunt.

Atrial septal defect in a man aged 20 who died


of a ruptured angiomatous malformation in the
cerebellum. Note the right ventricular
enlargement. No recorded cardiac symptoms.

Discuss pathogenesis of pulmonary hypertension and shunt reversal

Obstructions:
Coarctation of aorta
Congenital aortic valve stenosis
2013 - Cardiovascular IV – Valvular heart disease, infective endocarditis & congenital heart disease
Page 7
CARDIOVASCULAR IV
Demonstration pots
Royal Brisbane Clinical School

Woman aged 73 who died of metastatic


gall-bladder carcinoma. There were
recent infarcts in kidney, spleen, brain
and heart. Thrombi were also present in
the femoral veins and the inferior vena
cava.

Woman aged 81 admitted with 3 week history


of rigors, fever and polyarthritis. Developed
aortic diastolic murmur 1 week later, and
escherichia coli was grown from blood and
joint aspirate. Died 2 weeks after admission
with severe heart failure. Right pelvi-ureteric
stenosis and pyonephrosis found at autopsy.
2013 - Cardiovascular IV – Valvular heart disease, infective endocarditis & congenital heart disease
Page 8
2013 - Cardiovascular IV – Valvular heart disease, infective endocarditis & congenital heart disease
Page 9
CARDIOVASCULAR IV
Demonstration pots
PA/Mater Clinical School

Aortic valve prosthesis inserted in 1965, and


longterm anticoagulant therapy instituted. In
October, 1973, developed subdural haematoma
after minor trauma. As a result, anticoagulant
therapy was given only intermittently, and patient
developed symptoms suggestive of multiple
episodes of cerebral embolism. At autopsy in
1974, thrombosis was found to have occurred on
the prosthesis, and there were cerebral and
renal infarcts of various ages.
2013 - Cardiovascular IV – Valvular heart disease, infective endocarditis & congenital heart disease
Page 10

Man aged 53 with metastatic


carcinoma of the caecum.

Man of 68.

Woman aged 50; an


incidental autopsy finding.
2013 - Cardiovascular IV – Valvular heart disease, infective endocarditis & congenital heart disease
Page 11

Woman aged 63 who died of myocardial


infarction.
2013 - Cardiovascular IV – Valvular heart disease, infective endocarditis & congenital heart disease
Page 12
CARDIOVASCULAR IV
Demonstration pots
Ipswich Clinical School

Woman aged 35.

Man aged 48.


2013 - Cardiovascular IV – Valvular heart disease, infective endocarditis & congenital heart disease
Page 13

Man aged 56 who died suddenly and


unexpectedly. There was a past history of
syncopal attacks.

Woman aged 55 who died 3 days after the


onset of rigors and generalized myalgia.

Patient aged 24. The artery at the bottom left


is the femoral.

Man aged 65.


2013 - Respiratory I - Pathology of Pneumonia
Page 1
MBBS2 Tutorial – Respiratory I – Pathology of Pneumonia

***Important note: “The students are strongly advised to attend/listen to the relevant week’s pathology
lecture prior to attending the tutorials as all the conditions may not be covered in the tutorials and any
questions may be addressed in the discussion forum”

Learning Objectives
Understand and recognize the pathology and complications of pneumonia.
Understand the pathogenesis of tuberculosis.
To recognize the features of tuberculosis

Reproduced from Robbins & Cotran Pathologic


Basis of Disease 7th Ed. 2005

Histology - Normal Lung. Note


the thin alveolar septa (arrow)
containing only a single capillary
and thin investing connective
tissue support.

Pneumonia:
Types – broncho and lobar
Pathogenesis of each type
stages of lobar pneumonia
complications of pneumonia
2013 - Respiratory I - Pathology of Pneumonia
Page 2

Acute bronchopneumonia. Small grey foci of


consolidation (blue arrows) are widely
scattered through both lungs, which are
hyperaemic. Black arrows indicate pre-
existing carbon deposition.

Acute, pneumococcal lobar pneumonia.


A young woman who died within 24
hours of admission to hospital in spite of
antibiotic therapy. Arrows indicate the
area of consolidation.

Special pneumonias

Viral and mycoplasma


Legionnaire’s
Lung infections in immune-compromised host:
1. Pneumocystis carinii
2. Cytomegalovirus
3. Aspergillus
4. Candida
5. other fungi

Chronic lung infections

Tuberculosis
-organism
- granulomatous reaction pattern
-typical histology

Fibrocaseous tuberculosis. Note


zones of caseous necrosis (N),
epithelioid macrophages (E), giant
cells (arrows).
2013 - Respiratory I - Pathology of Pneumonia
Page 3

ZN stain

Acid fast bacilli


(arrows) (Ziehl-Neelsen
stain).

-primary, secondary, progressive (miliary)

Reproduced from Robbins &


Cotran Pathologic Basis of
Disease 7th Ed. 2005

Primary TB
- Ghon focus
- Ghon complex
Secondary TB
- location and histology
Progressive pulmonary tuberculosis
- pathogenesis of miliary TB

Lung abscess

Definition
Causes and mechanism:
1. aspiration
2. pre-existing lung infection eg pneumonia
3. immunosuppression
4. septic emobolism to lung
5. neoplasia
2013 - Respiratory I - Pathology of Pneumonia
Page 4

Woman aged 24 with Staphylococcus aureus septicaemia. Three


weeks before death a boil on her buttock had been incised, and a few
days later she developed fever and myalgia. Her condition then
deteriorated rapidly: multiple pustules appeared in the skin, she
became dyspnoeic and deeply jaundiced, urine volume fell and there
was clouding of consciousness. White cell count was 19.6 x 109/l,
with 84% neutrophils. Terminally, there was evidence of
disseminated intravascular coagulation.

Reproduced from Robbins & Cotran


Pathologic Basis of Disease 7th Ed.
2005
2013 - Respiratory I - Pathology of Pneumonia
Page 5
RESPIRATORY I
Demonstration pots
Royal Brisbane Clinical School

Normal lung from a young


adult

Man aged 41 who died 4 days after


the onset of a cough productive of
brownish sputum and of sharp left-
sided chest pain.

Woman aged 24 with Staphylococcus aureus septicaemia. Three weeks before death
a boil on her buttock had been incised, and a few days later she developed fever and
myalgia. Her condition then deteriorated rapidly: multiple pustules appeared in the
skin, she became dyspnoeic and deeply jaundiced, urine volume fell and there was
clouding of consciousness. White cell count was 19.6 x 101, with 84% neutrophils.
Terminally, there was evidence of disseminated intravascular coagulation.
2013 - Respiratory I - Pathology of Pneumonia
Page 6

Woman aged 23 years who died 2 weeks after the onset of


symptoms of respiratory infection. She developed an
erythematous rash, peritonitis, pericarditis and shock in the
terminal stages of her illness. The organism was isolated from
sputum, pleural fluid and blood. Antibiotics to which the
organism was sensitive were used only on the last day of life.

Woman aged 32 who died 5 days after the onset of


cough, chest pain and fever. She received intensive
antibiotic therapy during the last 3 days of life.

Man aged 70 who died 2 weeks after removal of a


meningioma. He remained unconscious following
the operation, and autopsy disclosed extensive
cerebral infarction.

.
2013 - Respiratory I - Pathology of Pneumonia
Page 7

RESPIRATORY I
Demonstration pots
PA/Mater Clinical School

Man aged 72 years.

Woman aged 32 who died 5 days after the onset of


cough, chest pain and fever. She received intensive
antibiotic therapy during the last 3 days of life.
2013 - Respiratory I - Pathology of Pneumonia
Page 8

Woman aged 41 with renal failure due to amyloid disease.


The specimen comprises the lower lobe of the left lung.

Aged 12 months.
2013 - Respiratory I - Pathology of Pneumonia
Page 9
RESPIRATORY I
Demonstration pots
Ipswich Clinical School

Male alcoholic aged 61 who had been drinking heavily


and who had been sick for several weeks before death.

Man aged 76 with Parkinson’s disease..

Man aged 52 who died of staphylococcus aureus


septicaemia complicating a shoulder injury.
Abscesses were also present in the kidneys and
spleen.
2013 - Respiratory I - Pathology of Pneumonia
Page 10

Man aged 65. There was a two month of lethargy, weight


loss, fever and dry cough.

Girl aged 4 years with fibrocystic disease of the pancreas.


2013 - Respiratory System II
Page 1
MBBS2 Tutorial - Respiratory System II
Chronic Lung Disease and Diseases of Pulmonary Vasculature

***Important note: “The students are strongly advised to attend/listen to the relevant week’s pathology
lecture prior to attending the tutorials as all the conditions may not be covered in the tutorials and any
questions may be addressed in the discussion forum”

Learning Objectives:

1. To understand the mechanisms leading to chronic lung disease


2. To understand the differences between obstructive and restrictive lung disease
3. Gross and histological features of the different forms of diffuse lung disease
4. Knowledge of the causes and complications of chronic lung disease
5. To know the important diseases of pulmonary vasculature

Obstructive lung disease

Includes emphysema, chronic bronchitis, asthma and bronchiectasis.

Emphysema
Definition & types
pathogenesis
morphology
Clinical course

Reproduced from Robbins &


Cotran Pathologic Basis of
Disease 7th Ed. 2005

Reproduced from Robbins &


Cotran Pathologic Basis of
Disease 7th Ed. 2005
2013 - Respiratory System II
Page 2

Reproduced from Robbins &


Cotran Pathologic Basis of
Disease 7th Ed. 2005

Severe panacinar emphysema in a man aged


85. Here the extent of tissue destruction is
greater. There is no fibrosis.

Chronic Bronchitis
Definition
pathogenesis
morphology
Clinical features

Asthma
Definition
Pathogenesis
Types – atopic, non-atopic, drug induced and occupational
Morphology
Clinical course

Reproduced from Robbins &


Cotran Pathologic Basis of
Disease 7th Ed. 2005

Left. Microscopically, the major change is


eosinophil infiltration into the wall of the bronchi.
Right. Higher magnification of the bronchial wall
showing the large number of eosinophils (arrows)
2013 - Respiratory System II
Page 3
Bronchiectasis
Definition
Pathogenesis
Morphology

X3360B. Death due to acute


bronchopneumonia in a patient
with bronchiectasis.

Restrictive or diffuse interstitial lung disease

Categories : fibrosing eg usual interstitial pneumonia (UIP), pneumoconiosis


Granulomatous eg sarcoidosis
Other eg pulmonary alveolar proteinosis
Pneumoconiosis

Important complications of chronic lung disease:

UIP: Reproduced from


Robbins & Cotran Pathologic
Basis of Disease 7th Ed. 2005

Pulmonary asbestosis in a man aged 50 who was exposed to


asbestos dust 16 years before death. He had marked dyspnoea and
clubbing of fingers during the last 6 years of life.
Fibrocaseous tuberculosis is also present in upper lobe (red
arrows). Note that fibrosis in asbestosis is maximal in lower zones
(black arrows).On chest X-rays the changes of tuberculosis are
seen in the upper lobes while those of asbestosis are seen in the
lower lobes.
2013 - Respiratory System II
Page 4

(Left). Asbestosis. In this low power view normal


alveoli are seen at the top of the picture (red arrows)
and interstitial fibrosis in the lower part. (black
arrows). See the enlarged air spaces which
contribute to the honeycomb appearance.
(Right). High power view shows asbestos bodies
(black arrows).

Calcified fibrous plaques in the parietal pleura of a man aged


63 who was exposed to asbestos dust 25 years before death.
These plaques do not contain asbestos bodies

Sarcoidosis: Reproduced
from Robbins & Cotran
Pathologic Basis of Disease
7th Ed. 2005

Malignant pleural mesothelioma in a man aged 63 who was


exposed to asbestos dust 25 years before death.
2013 - Respiratory System II
Page 5
Diseases of Vascular Origin

Broad classification
1. damage to vessels eg arteritis
2. obstruction eg pulmonary emboli
3. variations in intravascular pressure eg pulmonary hypertension

Cor pulmonale in a man aged 64. See the thickened right


ventricular wall (red arrows). He had been treated for
chronic obstructive pulmonary disease for 14 years before
death. He had smoked 60 cigarettes a day for 40 years. At
the time of his final admission to hospital, the pH of his
arterial blood was 7.16

Reproduced from Robbins & Cotran


Pathologic Basis of Disease 7th Ed.
2005
2013 - Respiratory System II
Page 6
RESPIRATORY SYSTEM II
Demonstration Pots
Royal Brisbane Clinical School

Eighteen year old man with 6 months' history Of


wheezing

80 yrs/M chronic smoker

85yrs/M

59 yrs/M cement factory worker and who suffered from


respiratory insufficiency
2013 - Respiratory System II
Page 7

73 yrs/Male complicated by right heart failure.


Plasma bicarbonate 38 mmoll.

51 yrs /M smoker with Eisenmenger's syndrome


complicating atrial septal defect.
2013 - Respiratory System II
Page 8
RESPIRATORY SYSTEM II
Demonstration Pots
PA/Mater Clinical School

72 yrs /M. Culture of the bronchial exudate


grew Pseudomonas aeruginosa.

Lung from a young boy


2013 - Respiratory System II
Page 9

Parietal pleura 78 yrs/M

Pulmonary artery from a


young woman
2013 - Respiratory System II
Page 10
RESPIRATORY SYSTEM II
Demonstration Pots
Ipswich Clinical School

25 yrs/F. She had hypertrophic pulmonary


osteoarthropathy and cor pulmonale.

67 yrs/M with a long history of repiratory insufficiency


and corpulmonale. shortly before death arterial blood
PO2 was 37mm Hg and PCO2 43mm Hg.

83 yrs/M
2013 - Respiratory System II
Page 11
2013 - Respiratory System III
Page 1
MBBS2 Tutorial – Respiratory System III
Pulmonary Tumours and Diseases of the Upper Respiratory Tract

Learning Objectives:

1. Classification of pulmonary tumours (including mesothelioma).


2. Recognized appearances and complications of lung tumours
3. Understand the principal pathologies of the upper aerodigestive tract, particularly carcinoma of
the oropharynx and larynx

Pulmonary tumours

Primary or secondary

Primary:

Aetiology and Pathogenesis

Like other cancers get stepwise accumulation of genetic abnormalities that transform benign bronchial
epithelium to neoplastic tissue. Major environmental insults that inflict genetic damage are known

Precursor lesions – does not imply that progression to invasion will occur
Types – 90-95% carcinomas
- bronchial carcinoids
- mesenchymal tumours

Classification of carcinomas:
- classified according to histological appearance
- squamous cell carcinoma (25%-40%)
- adenocarcinoma (25%-40%)
- small cell (20%-25%)
- large cell (10%-15%)

Clinically classified into two groups on the basis of likelihood of metastases and response to treatment -
small cell and non-small cell

Morphology : most tend to arise centrally, adenocarcinoma tends to arise peripherally.


Others – harmatoma (coin lesion)
Complications
Paraneoplastic syndrome
Staging
2013 - Respiratory System III
Page 2

Squamous, adeno, small cell and


large cell carcinoma. Reproduced
from Robbins & Cotran Pathologic
Basis of Disease 7th Ed. 2005

Broncho alveolar carcinoma.


Reproduced from Robbins &
Cotran Pathologic Basis of Disease
7th Ed. 2005

Carcinoid tumour (red arrow)


with obstruction, bronchiectasis
(blue arrow) and infection.
Operative surgical specimen.
2013 - Respiratory System III
Page 3

Numerous metastases from renal cell


carcinoma. Reproduced from
Robbins & Cotran Pathologic Basis
of Disease 7th Ed. 2005

Permeation of peribronchial pulmonary lymphatics by metastatic


gastric carcinoma. This is often referred to as lymphangitis
carcinomatosa. This pattern of spread may be caused by a number
of different tumours, including carcinoma of the lung itself.
Note the peri lobular (red arrows) and peribronchial (blue arrows)
distribution.

Pleural tumours – mesothelioma

Upper Respiratory Tract

Larynx
Inflammations :
– epiglottitis
2013 - Respiratory System III
Page 4

Acute epiglottitis. Note the oedema and


congestion (arrows).

Croup (LTB)

Acute laryngo-tracheo-bronchitis in a man aged 48 with alcoholic cirrhosis


and bleeding oesophageal varices who was comatose during the last few days
of life. Note the reddening of the epiglottis and the tracheal mucosa.

Reactive nodules

Papilloma

Carcinoma of larynx:

Reproduced from Robbins & Cotran


Pathologic Basis of Disease 7th Ed.
2005
2013 - Respiratory System III
Page 5
RESPIRATORY SYSTEM III
Demonstration Pots
Royal Brisbane Clinical School

Operative surgical specimen of a 50 year old


man who is a chain smoker and presented
with chronic cough.

Operative surgical specimen from 66 year


old man, a chronic smoker, presented with
chronic cough with expectoration.

A 70 year old man with chronic


cough and significant weight
loss.

A female epileptic aged 57 years was eating in


bed, and suddenly moaned and fell backwards
unconscious. When an ambulance bearer
arrived she was found to be dead.
2013 - Respiratory System III
Page 6

Operative surgical specimen from


62 year old female who presented
with hoarseness of voice and
dyspnea.

Lungs of a 65 year old man who


underwent a gastrointestinal surgery
few years back.

Man aged 75 who suffered a cardiac arrest


associated with myocardial infarction 7 days
before death, and remained deeply unconscious
thereafter.

A man aged 56 presented with mental confusion caused by severe


hypercalcaemia (4.05 m mol/l). He died soon after admission.

A man aged 63 who was exposed to asbestos dust 25 years presented


with dyspnea before death.
2013 - Respiratory System III
Page 7
RESPIRATORY SYSTEM III
Demonstration Pots
PA/Mater Clinical School

A man aged 48, non smoker, with


breathlessness and cough.

Autopsy specimen from a 67 year old man who


complained of hoarseness of voice and lately
dyspnea.

4 yr old girl c/o sore throat at night. Death


occurred during the night.
2013 - Respiratory System III
Page 8

Lung of a woman aged 37 who


underwent breast surgery 3 years ago.

A 70 year old chronic smoker


presented with chronic cough & weight
loss.

A man aged 53 presented 6 months before death


with cough productive of purulent sputum and
chronic tiredness. He had smoked more than 20
cigarettes a day for over 20 years.

Operative surgical specimen from a man aged 69


who had smoked heavily for many years with
dyspnea, cough & fever.
2013 - Respiratory System III
Page 9

A man aged 63 who was exposed to asbestos dust for


25 years presented with breathlessness before death.
2013 - Respiratory System III
Page 10
RESPIRATORY SYSTEM III
Demonstration Pots
Ipswich Clinical School

Man aged 72 with staphylococcus aureus


septicaemia and mutliple organ failure.

Male aged 66 with chronic bronchitis.


Smoked 30 cigarettes a day for many years.

Operative surgical specimen from a 41 year


old man who presented with hoarseness of
voice.
2013 - Respiratory System III
Page 11

Lung of a 69 year old man who underwent a


cutaneous surgery 4 years before death.

A man aged 48 with a history of exposure to


asbestos dust for 25 years presented with
breathlessness before death.

Lung in a man aged 53 who had smoked 30


cigarettes a day for many years. Death was
due to cerebral embolism secondary to left
ventricular mural thrombosis complicating
acute myocardial infarction.
2013 - Pathology of the Liver
Page 1
MBBS2 - Pathology of the Liver

Please note these objectives are a guide to what students are expected to understand after the
lectures, tutorials and private reading.

Learning objectives from lectures and tutorials


1. To know the different types of liver injury that can be seen in alcoholic liver disease
2. To understand the causes, pathogenesis and complications of cirrhosis
3. To know the infections that result in acute and chronic liver disease
4. To recognize the types of liver tumours
5. To be aware of the major vascular and intrahepatic biliary diseases

Review the histology of the liver and blood supply – acinar and lobular concepts. Acinar concept is
considered more useful as it explains better many of the pathophysiological disturbances in liver
disease.

Reproduced from Robbins &


Cotran Pathologic Basis of Disease
7th Ed. 2005.

Reproduced from Robbins &


Cotran Pathologic Basis of Disease
7th Ed. 2005.

Liver disease – Primary – alcoholic liver disease, metabolic liver disease, drug-induced liver disease,
infective liver disease, hepatocellular carcinoma
Secondary – CCF, Metastatic disease

Role of liver biopsies in pathology.

Functional reserve is huge therefore masks clinical impact of early liver disease.
So tends to be a slowly progressive pattern.
Patients usually present with chronic disease.

Patterns of hepatic injury:


Limited number of responses to injury.
2013 - Pathology of the Liver
Page 2
5 general responses:
- inflammation
- degeneration and intracellular accumulation

X2904B. Marked fatty change of the liver.


Man aged 45 who was found in a confused
state in a boarding house room: he was
surrounded by empty wine bottles. Death
occurred as a result of inhalation of vomitus
soon after his admission to hospital. At the
time of autopsy it would have felt very greasy.
Note the pallor of this specimen.

Fatty liver histologically shows clear spaces in


the hepatocytes. Individual cells are swollen
with the fat (it is clear because fat is removed
from the tissues during processing) and this is
the reason that fatty livers are usually enlarged
clinically. Cytoplasm and nucleus are
displaced to the periphery of the cell by the
lipid droplet.

- necrosis and apoptosis


- regeneration
- fibrosis
Discuss each response

The features and pathogenesis of:


Hepatic failure
Cirrhosis

What are the defining features of cirrhosis?


Various aetiological agents (macroscopic and often microscopic examination may not be diagnostic).
Aetiological agents:
1. Alcohol, other toxins and drugs
2. Inflammatory conditions eg. viral, autoiummune
3. Biliary disorders
4. Inherited diseases eg. Haemochromatosis
5. Vascular diseases eg. Hepatic vein thrombosis
6. Cryptogenic

Micronodular (most likely alcohol or haemochromatosis) vs macronodular


2013 - Pathology of the Liver
Page 3

X164B. Coarsely nodular cirrhosis in a


female aged 12 years who had suffered
from acute hepatitis 5 years before death.
The mode of evolution of the cirrhosis was
not followed by liver biopsy in this case.
See the large rounded nodules (arrows)
each surrounded by fibrous tissue.

Reproduced from Robbins &


Cotran Pathologic Basis of Disease
7th Ed. 2005

Complications:
1. Portal hypertension – why does it occur with cirrhosis?

Portal hypertension

Reproduced from Robbins &


Cotran Pathologic Basis of Disease
7th Ed. 2005
2013 - Pathology of the Liver
Page 4

X3710. Large oesophageal varices


(arrows) in a patient with chronic
portal hypertension. They have
partially collapsed in the fixed
specimen.

2. Hepatic failure
3. Hepatocellular carcinoma

Congenital liver disease

X308B. Polycystic disease of liver in


a woman aged 51 who died of
uraemia secondary to polycystic
kidneys (autosomal dominant, or
"adult", type.

Alcoholic liver disease

Reproduced from Robbins & Cotran


Pathologic Basis of Disease 7th Ed.
2005

Discuss the morphology and pathogenesis


2013 - Pathology of the Liver
Page 5

This photomicrograph shows Mallory’s hyaline (M),


hepatocyte swelling (H)
and neutrophil infiltration (N).
Although Mallory’s hyaline is the best known
histological feature, it may not be present in every case or
it may be difficult to find. The fibrosis is important; it
develops in response to the inflammation and begins
around the central vein. With repeated deposition it
eventually links the central veins to the portal tracts,
causes distortion and leads to cirrhosis.

Alcoholic cirrhosis. This shows parenchymal nodules


(N) surrounded by fibrous septae (F). By the time
cirrhosis has developed, the features of alcoholic liver
disease may no longer be detectable. This is particularly
the case if the patient has stopped drinking. Obviously
if fatty change and Mallory’s hyaline are seen, then
alcohol becomes a very likely aetiology.

Metabolic liver disease

NASH

Haemochromatosis – primary and secondary

Histology: Genetic haemochromatosis. This picture


shows iron as brown granular pigment (arrows) in
the hepatocytes. The portal tracts are expanded by
fibrosis (F), but the liver is not cirrhotic. In
haemochromatosis, the iron deposition occurs
primarily in the hepatocytes. This differs from
transfusional overload where the deposition is also
marked in reticuloendothelial cells (macrophages),
particularly the Kupffer cells.
2013 - Pathology of the Liver
Page 6

Iron is detected in the cells using


Prussian blue, or Perls’ stain,
which stains the iron blue. This is 4
+ iron deposition.

Wilsons disease

Drug-induced liver disease

At least 10% of drug reactions involve the liver, as the liver plays an important role in metabolism and
in the conjugation and elimination of toxic substances from the body.

Two types : predictable (dose-related) and unpredictable (idiosyncratic)

Two sites of damage – hepatocellular or cholestatic.

Histology:drug-induced
intrahepatic cholestasis. Note
perivenular intracanalicular
cholestasis (black arrows)

Infective liver disease


The liver may be affected by a wide range of pathogens, including viruses, bacteria, parasites, fungi
and rickettsia.
2013 - Pathology of the Liver
Page 7
1. Viral
Viruses, such as Hepatitis A, B, C, and others, may cause a range of changes in the liver. These
include a very mild and subclinical hepatitis, chronic hepatitis and even fulminant and rapidly fatal
hepatitis. As well as the hepatitis virus, others such as cytomegalovirus, Epstein-Barr and herpes
simplex can cause a hepatitis illness in addition to effects in other parts of the body.

A-C. Viral hepatitis. Picture A shows slight


disarray of the hepatocyte plates, an infiltrate of
lymphocytes in the lobules, and some apoptotic
bodies better seen in B (arrowed) and C.
Apoptotic cell death is the characteristic
mechanism of deletion of virally-infected cells.

Picture D shows the typical appearance of chronic


hepatitis, and the important features are portal
inflammation,(I) portal fibrosis (this does not
develop in every case) and interface hepatitis (so-
called "piecemeal necrosis"), which is arrowed. This
last change is characterised by irregularity of the
limiting plate, extension of lymphocytes into the
adjacent parenchyma and apoptotic bodies due to
hepatocyte death. With time the development of
fibrosis may evolve into cirrhosis, so chronic
hepatitis is an important lesion.
2013 - Pathology of the Liver
Page 8

Image E shows ground glass inclusions in hepatocytes


(arrowed). These are cytoplasmic collections of hepatitis B
surface antigen. This is how they appear in H&E stained
sections. They stain positively with immunoperoxidase stains
for Hep B antigen.

2. Bacterial

X2344A. Large multiloculated liver abscesses


(arrows) in a woman aged 57 who died 6 weeks after
cholecystectomy and exploration of the common bile
duct. Much of the pus that filled the abscess cavities
has been lost during preparation of the specimen for
mounting.

X2438. Disseminated tuberculosis of the liver in a


female aged 6 years. Circumscribed caseous nodules
(arrows) surrounded by tuberculous granulation tissue
are scattered through the parenchyma. Some are bile
stained, presumably as a result of involvement of bile
ducts by the necrotising process. Histological
examination showed that numerous tubercles were
also scattered through the liver: these are difficult to
see with the naked eye. Autopsy disclosed extensive
tuberculosis of the lungs and abdominal viscera.

3. Parasitic and helminthic


2013 - Pathology of the Liver
Page 9

X2175B. Hydatid disease of the liver in a woman


aged 36 who presented with colic due to the presence
of daughter cysts in the biliary passages. Partial
hepatectomy specimen. This is the larval stage of the
dog tapeworm E. granulosus.

Vascular liver disease


The liver has a dual blood supply, and this protects against infarction in many cases. Nevertheless, the
circulation may be affected in a number of ways.

Reproduced from Robbins &


Cotran Pathologic Basis of
Disease 7th Ed. 2005

Examples :

1. impaired flow to the liver eg. Portal vein thrombosis

2. intrahepatic impaired flow – chronic venous congestions of the liver causing a ‘nutmeg liver’
eg. in longstanding heart failure

3. outflow obstruction: eg. Budd-Chiari syndrome – thrombosis of the major hepatic veins causes
extreme blood retention in the liver

Neoplasia
The liver is a common site for metastatic spread. Primary liver cancers are relatively rare in Western
populations but are common in countries with a high incidence of hepatitis B. There are several factors
known to be important for increasing the rate of liver cancers eg. Hepatitis B and C, aflatoxin and
cirrhosis
2013 - Pathology of the Liver
Page 10

Classification of major liver tumours and tumour-like conditions:


Focal nodular hyperplasia
Haemangioma
Liver cell adenoma
Bile duct adenoma
Hepatoblastoma
Hepatocellular carcinoma
Metastatic

Intrahepatic biliary diseases


Bile stasis can occur at any level from the hepatocyte and canaliculus to the extrahepatic bile ducts.
Diseases affect the intrahepatic biliary apparatus at different levels.

1. canaliculus – drug induced cholestasis (already discussed)


2. small intrahepatic bile ducts – primary biliary cirrhosis

Primary biliary cirrhosis in a woman aged


56. Ten year history of itch. Smooth muscle
and mitochondrial antibodies present in
blood. Serum alkaline phosphatase
persistently high. Died with liver failure and
pneumonia. Extrahepatic bile ducts found to
be normal at autopsy. While the aetiology
cannot be determined macroscopically here,
biliary tract disease is suggested as the liver
has a greenish tinge due to bile stasis.

Portal inflammation and lack of


bile ducts in primary biliary
cirrhosis.

3. large intrahepatic bile ducts – primary sclerosing cholangitis

Concentric peribiliary fibrosis


(arrows) in primary sclerosing
cholangitis.
2013 - Pathology of the Liver
Page 11

4. neoplasia – bile duct adenoma


- cholangiocarcinoma

Clonorchis sinensis infection and


mucus-secreting cholangiocarcinoma in
a Chinese male aged 55 from Hong
Kong. There are many flukes in the
intrahepatic bile ducts. (some of the
flukes are presented in the separate
circle).
2013 - Pathology of the Liver
Page 12
PATHOLOGY OF THE LIVER
Demonstration Pots
Royal Brisbane Clinical School

X2838. Normal liver from a woman aged 37


who was killed in a motor vehicle accident.

Man aged 86 with multiple myeloma who


received many blood transfusions..

Man aged 43 known to have had a heavy alcohol


intake over many years. Also had hypertensive heart
failure.
2013 - Pathology of the Liver
Page 13

Woman aged 35 with history of hepatitis . At the time of death


had ascites, liver failure and bronchopneumonia. What is the
other organ present and how is it affected?

Woman aged 30 with a one month history of anorexia,


nausea, vomiting and abdominal discomfort. Three weeks
before death she noticed jaundice, dark urine and pale stools.

Man aged 47 with long standing heart


failure due to aortic valve disease.

Man aged 41. Death was due to GI


haemorrhage.
2013 - Pathology of the Liver
Page 14
PATHOLOGY OF THE LIVER
Demonstration Pots
PA/Mater Clinical School

Normal adult liver. Because it has been necessary to


perfuse the liverwith formalin to achieve adequate
fixation, the liver tissue has become alittle paler than
that seen at operation or autopsy.

Patient had a history of jaundice as a child.

The patient had chronic myeloid leukaemia, and received many


blood transfusions for anaemia.
2013 - Pathology of the Liver
Page 15

History of heart failure.

An incidental autopsy finding in a man aged


53 who died of cerebral haemorrhage.

Man aged 73.


2013 - Pathology of the Liver
Page 16
PATHOLOGY OF THE LIVER
Demonstration Pots
Ipswich Clinical School

Man aged 67 with severe heart failure.

Man aged 38.

The pale colour suggests the condition, but a definitive diagnosis


cannot be made without histological examination. The patient was a
67 year old obese female diabetic who died of recurrent pulmonary
embolism: the diabetes was poorly controlled during her terminal
illness.
2013 - Pathology of the Liver
Page 17

Female aged 12 years who had suffered


from acute hepatitis 5 years before
death.

Man aged 73 with history of heavy alcohol intake until 4


years before death, which followed the development of
ascites, confusion and drowsiness. Note the small nodule
that differs from the rest – what is this?

Chinese male aged 55 from Hong Kong. There


are many flukes in the intrahepatic bile ducts.
2013 - Pathology of the Liver
Page 18

An incidental autopsy finding in a


woman who died of a ruptured
aneurysm of the internal carotid artery.

Woman of 39 who had had many transfusions for


anaemia over a period of 10 years. The nature of the
anaemia is uncertain: erythrocyte survival was shown
to be reduced but red cell production was also
depressed.
2013 – Upper Gastrointestinal Tract
Page 1
MBBS2 Tutorial –Upper Gastrointestinal Tract

***Important note: “The students are strongly advised to attend/listen to the relevant week’s pathology
lecture prior to attending the tutorials as all the conditions may not be covered in the tutorials and any
questions may be addressed in the discussion forum”

Learning Objectives:
1. To recognize that inflammation and tumours are the major diseases of the mouth, salivary
glands
2. To discuss and recognize the prinicipal infective, inflammatory and neoplastic pathologies of
the oesophagus and stomach, particularly reflux oesophagitis, peptic ulcer disease and
carcinoma

Salivary glands

Sialadenitis – inflammation of the salivary gland


May be viral eg mumps
Bacterial – secondary to dehydration or obstruction
Autoimmune – Sjogrens syndrome
Sialolithiasis
Neoplasms

Pleomorphic adenoma of the parotid in a man aged


25. Operative surgical specimen

A pleomorphic adenoma with glandular spaces lined


by cuboidal epithelial cells (arrows), groups of
smaller darker myoepithelial cells (short arrows)
and an abundant myxoid stroma.

Diseases of Oral Mucosa

Infections : viral - Herpes


Bacterial
Fungal – Candida

Inflammation : aphthous ulcers (many systemic conditions are associated with oral lesions) –
relationship to gastrointestinal disorders
2013 – Upper Gastrointestinal Tract
Page 2

Neoplasms:
Premalignant conditions:
1. leukoplakia

Reproduced from Robbins & Cotran


Pathologic Basis of Disease 7th Ed.
2005

2. erythroplakia

Squamous cell carcinoma:


Progression of oral cancer

(Reproduced from Robbins & Cotran Pathologic Basis of Disease 7th Ed. 2005)
2013 – Upper Gastrointestinal Tract
Page 3

Oesophagus
Congential lesions:
Atresia, usually associated with tracheo-oesophageal fistula

(Reproduced from Robbins & Cotran Pathologic Basis of Disease 7th Ed. 2005)

Acquired lesions:

1. Webs, rings and strictures

Stricture (arrows) of the lower end of the oesophagus in a


man aged 69 with a past history of hiatus hernia and
reflux oesophagitis.

2. Achalasia
2013 – Upper Gastrointestinal Tract
Page 4

Chronic achalasia of the oesophagus in an elderly man. The


obstruction is at the bottom right of the specimen. Note the
marked dilatation of the oesophagus and the hypertrophy of
its muscle.

3. diverticula
4. Mallory-Weiss tear

Mallory-Weiss tear (arrow) at the gastro-oesophageal junction in a


male alcoholic aged 40, who died of massive gastro-intestinal
haemorrhage

5. Oesophageal varices

Oesophageal varices (arrows)

6. Hiatus hernia – two types sliding (95%) and rolling (5%), associated with reflux. Complications
include strangulation and obstruction.

7. Infections:
Viral – Herpes
2013 – Upper Gastrointestinal Tract
Page 5

Herpes simplex. Characteristic histological features


are opaque 'ground glass' nuclei (arrows),
eosinophilic nuclear inclusion bodies and
multinucleate giant cell formation (arrowhead) in
infected squamous epithelium.

9. Gastro-oesophageal reflux

10. Barrett’s oesophagus

Barrett's oesophagus

long-standing Barrett's oesophagus with dysplasia

Malignant tumours

Squamous cell carcinoma (arrows) of the lower


oesophagus in a man aged 65: operative surgical
specimen.
2013 – Upper Gastrointestinal Tract
Page 6
Adenocarcinoma

Adenocarcinoma (arrows) of
the cardio-oesophageal
junction.

Adenocarcinoma arising in
Barrett's oesophagus.

Stomach
Difference between erosion and ulcer.

Acute gastritis and multiple acute ulcers (arrows) in


a man aged 72 with Staphylococcus aureus
septicaemia. There was multiple organ failure, but
the immediate cause of his death appeared to be
massive haemorrhage into the stomach.

Gastritis – H. pylori

Active chronic gastritis with Helicobacter


(arrowhead). Intraepithelial neutrophils are
indicated by arrow.
2013 – Upper Gastrointestinal Tract
Page 7

Silver stain showing Helicobacter in


superficial mucin.

Peptic ulceration

Role of H Pylori.

Chronic peptic ulcer on the lesser curvature of the


stomach with an eroded artery (arrow) in its base.
Man aged 67 who was known to have had a peptic
ulcer for 20 years. He had 3 haematemeses in the 3
days before partial gastrectomy was performed.
2013 – Upper Gastrointestinal Tract
Page 8

complications:

Perforated gastric ulcer


(arrow).

Malignant tumours of the stomach

Primary:

1. Adenocarcinoma – aetiology, types, complications. Krukenberg tumours.

Ulcerating carcinoma of the


stomach. Operative surgical
specimen.
2013 – Upper Gastrointestinal Tract
Page 9

Diffusely infiltrating carcinoma of the stomach


(a linitis plastica) in a woman aged 75.
Operative surgical specimen. Note the small,
thick walled stomach.

Signet ring cell type of carcinoma of stomach. This is the


common pattern seen in linitis plastica.
2013 – Upper Gastrointestinal Tract
Page 10
GIT I – UPPER GASTROINTESTINAL TRACT
Demonstration Pots
Royal Brisbane Clinical School

stomach

upper oesophagus.

male alcoholic aged 40

Stomach female aged 61.


2013 – Upper Gastrointestinal Tract
Page 11

stomach

partial gastrectomy specimen. Man aged 48 who had had symptoms of peptic
ulceration for many years

Bezoar with diverse components


removed surgically from the stomach of
a male aged 11 years.
lower oesophagus in a man aged 53.
Gastrectomy specimen. What
Carcinoma of the back 2013 – Upper Gastrointestinal Tract
effectofmight
the prednisone therapy
Page 12
Pharynx
tongue with deep central
have on an ulcer?
GIT I – UPPER GASTROINTESTINAL
ulceration. TRACT
Demonstration Pots
PA/Mater Clinical School
Deeply ulcerated gastric leiomyosarcoma
Gastrectomy
Localisedspecimen male aged
gastric mucosal 58.
thickening
due to hyperplasia of the mucus 2013 – Upper Gastrointestinal Tract
Page 13
secreting cells. This condition must
be distinguished from the rugal
thickening due to infiltrating
carcinoma.
upper oesophagus in a man aged 78.
2013 – Upper Gastrointestinal Tract
Stomach & Colon. Female aged 60
Trichobezoar Page 14
GIT I – UPPER GASTROINTESTINAL TRACT
Demonstration Pots
Ipswich Clinical School
Healed gastric ulcers. Small
Gastrectomy specimen: Male aged
Oesophagusleiomyoma.
partial gastrectomy male aged 67 2013 – Upper Gastrointestinal Tract
63. Page 15
2013 - GI III – Pathology of the Small Intestine & Exocrine pancreas
Page 1

MBBS2 Tutorial GIT III - Pathology of the Small Intestine

Learning Objectives

1. Know the major congenital abnormalities


2. To discuss the causes and pathology of ischaemic bowel disease
3. Recognize the pathology of the major causes of malabsorption
4. To discuss the major tumours of the small intestine
5. Understand the pathogenesis of appendicits and other pathologies of the appendix
6. To describe the causes, effects, macroscopic and microscopic features of acute pancreatitis
7. To describe the clinicopathological features of carcinoma of the pancreas

Small Intestine

Normal histology

Reproduced from Robbins & Cotran


Pathologic Basis of Disease 7th Ed.
2005

Congenital Abnormalities

Atresia and Stenosis – complete obstruction is called atresia, incomplete obstruction is stenosis.

Stenosis (black arrow) of second part of


duodenum in an infant. See the relation to the
pancreas (red arrow).The stomach is on the left.

Meckel’s diverticulum – located in ileum, result of incomplete regression of the vitello-intestinal duct.
Usually lined by small intestine mucosa. Heterotropic rests of pancreatic tissue or gastric mucosa may
be present (prone to ulceration and haemorrhage). Other complications – intussception, perforation and
incarceration.
2013 - GI III – Pathology of the Small Intestine & Exocrine pancreas
Page 2

Meckel's diverticulum. A woman aged 62. An


incidental autopsy finding. See the diverticulum
(arrows) arising from the anti mesenteric side of the
small intestine.

Meconium Ileus – small intestine obstruction resulting from viscous meconium in patients with cystic
fibrosis.

Meconium ileus and focal small gut stricture (arrow).


Specimen excised surgically from a female aged 2 days with
cystic fibrosis. Note the dilatation of the gut which is filled
with sticky, viscous secretions. This is one way in which cystic
fibrosis presents in a neonate. The stenosis is an unrelated
condition.

Ectopic pancreas

Nodule of ectopic pancreas (arrows) in the wall of the


small gut. Operative surgical specimen.
This may be asymptomatic, or may cause external
pressure induced obstruction or be the apex of an
intussusception.

Ischaemic bowel disease

4 main causes of acute vascular compromise to the bowel:


1. major arterial occlusion: usually superior mesenteric artery by thrombosis or embolism

Reproduced from Robbins & Cotran


Pathologic Basis of Disease 7th Ed.
2005
2013 - GI III – Pathology of the Small Intestine & Exocrine pancreas
Page 3
2. end artery occlusion: usually the result of vasculitis, producing small localized lesions, that may
perforate.

3. venous occlusion: hypercoaguable states, intraperitoneal sepsis

Haemorrhagic infarction of small intestine secondary


to venous obstruction. This shows full thickness
infarctio of the bowel wall. The red arrow indicates the
lumenal aspect, the black arrows the serosal aspect.

4. non-occlusive ischaemia: usually in the presence of atherosclerosis of the mesenteric arteries and
secondary to shock

Acute ischaemic enteropathy in a woman aged 63 with


metastatic leiomyosarcoma. She became shocked and
developed abdominal pain and tenderness with passage
of blood per rectum during the last few days of life. The
picture shows the lumenal aspect of the bowel.
Microscopy showed extensive mucosal necrosis and
ulceration, but reasonable preservation of the muscle
coats.

Chronic ischaemia:
Associated with mucosal ulceration and mural fibrosis mimicking inflammatory bowel disease and
often leads to a stricture.

Mechanical abnormalities leading to bowel obstruction


80% of bowel obstructions are due to hernias, intestinal adhesions, intussusceptions or volvulus.

Reproduced from Robbins & Cotran


Pathologic Basis of Disease 7th Ed.
2005

Malabsorption
2013 - GI III – Pathology of the Small Intestine & Exocrine pancreas
Page 4

Coeliac disease
Discuss aetiology and morphology

Normal small intestinal mucosal


biopsy showing finger-like villi and
villus length to crypt width ratio of
4:1.

Mucosal biopsy in coeliac disease


showing partial villous atrophy (villus
height (blue arrows) to crypt (black
arrows) length ratio of 1:1). See the
blunt villi and elongated crypts.

Mucosal biopsy in coeliac disease


showing total villous atrophy, crypt
(black arrows) hyperplasia and
increased inflammatory cells in the
lamina propria.

Higher magnification of (C) showing damaged


surface epithelium (black arrows) with loss of
brush border, vacuolation (red arrows) and
irregular arrangement of surface epithelial
cells, increased lymphocytes in the epithelium
(blue arrows) and large numbers of plasma
cells in the lamina propria.

Other causes of malabsorption eg. Tropical sprue, Whipples, Disaccharidase deficiency


2013 - GI III – Pathology of the Small Intestine & Exocrine pancreas
Page 5
Tumours

Classification:
Benign – adenomas, angiomas, lipomas, GIST
Malignant – Primary : adenocarcinoma, carcinoid, lymphoma, GIST
Metastatic

Carcinoid

Carcinoid tumour (black arrows) of the ileum in


a man aged 56: operative surgical specimen.

Small intestinal carcinoid. Note


grouping and regular appearance of
tumour cells.

Lymphoma

Diffuse malignant lymphoma (arrows) of the


small gut in a child aged 6 years. Operative
surgical specimen. Note how the tumour
extends to involve the bowel wall and the
adjacent mesentery.
2013 - GI III – Pathology of the Small Intestine & Exocrine pancreas
Page 6
Appendix
Acute appendicitis
Pathogenesis

Severe acute appendicitis in a female aged


14 years.

Acute appendicitis in a man aged 27:


operative surgical specimen.

Mucinous lesions
Mucocoele
Desciptive term to describe and appendix that is full of mucous. Usually benign but may result from an
adenocarcinoma.

Mucocele of the appendix.

Mucinous cystadenoma
Commonest mucinous neoplasm

Mucinous cystadenocarcinoma
2013 - GI III – Pathology of the Small Intestine & Exocrine pancreas
Page 7

Reproduced from Robbins &


Cotran Pathologic Basis of
Disease 7th Ed. 2005.

Pseudomyxoma peritonei

Pseudomyxoma peritonei arising from appendix. Male aged 52. This


is a condition in which the abdominal cavity is filled with mucus and
fibrosis. It results from spread to the peritoneum of an
adenocarcinoma from appendix, ovary or gut.

Carcinoid

Carcinoid tumour (arrows) at the tip of the


appendix(its commonest site) in a male aged
14 years: operative surgical specimen. Note
the characteristic yellow colour.

Pancreas

Retroperitoneal organ, the head lying within the duodenal loop, the body crossing the aorta and inferior
vena cava and the tail abutting onto the splenic hilum. It is a mixed endocrine and exocrine organ.

Pancreas from a man aged 32 who had been treated for


severe diabetes mellitus since early childhood. The
pancreas appears normal macroscopically.
2013 - GI III – Pathology of the Small Intestine & Exocrine pancreas
Page 8

Diseases of the pancreas can be divided into those affecting the exocrine pancreas and those affecting
the endocrine pancreas.

Exocrine pancreas

The main conditions to affect the pancreas are inflammation (acute and chronic pancreatitis) and
tumours.
Pancreatitis is divided into acute and chronic and is usually associated with acinar cell injury.

Acute pancreatitis

This is inflammation of the pancreas, usually associated with necrosis of acini and intrapancreatic
adipose tissue. It can be a mild self-limited disease or a serious disorder with a high mortality. Over
80% of cases are associated with either biliary calculi or alcohol abuse. Other less common causes
include: infection, ischaemia, drugs, trauma and metabolic causes eg hypercalcaemia.

The pathogenesis varies according to the cause, eg obstruction of the pancreatic duct by a gallstone
leading to damage to the epithelium of the duct and subsequently damage to the glands, as opposed to
ischaemia causing infarction and necrosis.

Relatively mild acute pancreatitis. The pancreas


appears mildly congested and foci of calcification
(arrows) are present within the gland.

Severe acute haemorrhagic pancreatitis. Note the obvious


haemorrhage and the widespread deposits of calcium (arrows).

Pathogenesis of the widespread deposits of calcium:

The main changes occurring in pancreatitis are:


Protease digestion of pancreatic substance
Elastase digestion of blood vessels with subsequent haemorrhage
Lipases digesting the fat

Accompanying inflammatory reaction.


2013 - GI III – Pathology of the Small Intestine & Exocrine pancreas
Page 9

Acute pancreatitis. The pre-existing


architecture has been essentially
destroyed.

Haemorrhage and necrosis of acute


pancreatitis, with some preserved pancreatic
parenchyma on the left (arrow).

Focal acute necrosis of perirenal fat in a patient who


had severe acute pancreatitis. The patient was oliguric
due to acute renal tubular necrosis of ischaemic type.
See the calcium deposition (arrows) highlighting foci of
enzymatic fat necrosis.

Carcinoma of the pancreas

Carcinoma of the pancreas generally refers to carcinoma of the exocrine pancreas. The great majority
of these arise from the ductal epithelium. The incidence of pancreatic cancer is increasing and as it
usually presents late, the prognosis is generally poor.

Carinoma (blue arrows) of the head of the pancreas with


obstruction of ducts and atrophy of the body (red arrow)
and the tail (black arrow).The dilated pancreatic duct can be
seen for the whole length of the pancreas.
2013 - GI III – Pathology of the Small Intestine & Exocrine pancreas
Page 10
GI III – PATHOLOGY OF THE SMALL INTESTINE
Demonstration pots
Royal Brisbane Clinical School

Operative surgical
specimen from a man who
had severe pain in right
iliac fossa.

Surgical specimen of
ileum from a man who
presented with intestinal
obstruction.

A male aged 3.5 months came with abdominal


pain, vomiting, blood in stools following an
episode of upper respiratory tract infection.
2013 - GI III – Pathology of the Small Intestine & Exocrine pancreas
Page 11

Operative surgical specimen of male


infant aged 2 days who had abdominal
distension & did not pass meconium after
birth.

An incidental surgical
finding in a man aged
78.

A man aged 72 presented 2 months


before death with epigastric pain, weight
loss and jaundice. Serum bilirubin was
94 mol/l and serum alkaline phosphatase
650 u/l.
2013 - GI III – Pathology of the Small Intestine & Exocrine pancreas
Page 12

A 66 year old woman came with c/o


abdominal pain, significant weight loss & &
recurrent episodes of localized thrombosis of
the superficial veins in the limbs.
2013 - GI III – Pathology of the Small Intestine & Exocrine pancreas
Page 13
GI III – PATHOLOGY OF THE SMALL INTESTINE
Demonstration pots
PA/Mater Clinical School

An incidental autopsy finding.

Small gut of an adult who had


tuberculous infection presented
with abdominal pain, vomiting &
blood in stools.

Ileum of a neonate who did not pass


meconium after birth and was born with
an APGAR score of 8.
1
2

2013 - GI III – Pathology of the Small Intestine & Exocrine pancreas


Page 14

Surgical specimen of ileum


from a man who presented with
intestinal obstruction.

Small gut from a man who was diagnosed to have


celiac disease many years ago was admitted with
abdominal pain & intestinal obstruction.

A 74-year-old man presented with an episode


of lower gastrointestinal (GI) bleeding and
progressively worsening anemia. His past
medical history includes surgical resection of
two separate cutaneous lesions on his back.

Surgical peritoneal specimen from a man who came


with an acute abdomen and on laparotomy was found
to have “jelly-belly” & a perforated appendix.
2013 - GI III – Pathology of the Small Intestine & Exocrine pancreas
Page 15

A 60 year old chronic alcoholic


presented with sudden onset of
severe abdominal pain that radiated
to back.

A 70 year old chronic smoker c/o


weight loss, malaise, abdominal
pain & recurrent episodes of
localized thrombosis of the
superficial veins in the limbs and
trunk.
2013 - GI III – Pathology of the Small Intestine & Exocrine pancreas
Page 16
GI III – PATHOLOGY OF THE SMALL INTESTINE
Demonstration pots
Ipswich Clinical School

An incidental autopsy finding in a


man aged 82.

Operative surgical specimen of male infant


aged 2 days who had abdominal distension
& did not pass meconium after birth.

A 67-year-old woman presented with


an episode of lower gastrointestinal (GI)
bleeding and progressively worsening
anemia. Her past medical history
includes surgical resection of two
separate cutaneous lesions on his
back.

A male aged 3.5 months came with


abdominal pain, bilious vomiting, red
currant jelly stools following an episode
of upper respiratory tract infection.
2013 - GI III – Pathology of the Small Intestine & Exocrine pancreas
Page 17

Operative surgical specimen of a male


aged 21 years who presented with pain
in right iliac fossa.

Operative surgical specimen of


terminal ileum from a 43 year old
man causing small gut obstruction.
He also had hepatomegaly, diarrhea,
and flushing.

A 66 year old woman who was a chronic


smoker came with c/o abdominal pain,
significant weight loss & recurrent
episodes of localized thrombosis of the
superficial veins in the limbs.
2013 - GI III – Pathology of the Small Intestine & Exocrine pancreas
Page 18

A 60 year old chronic


alcoholic presented with
sudden onset of severe
abdominal pain that radiated
to back.
2013 Case Study Questions

CASE 1
a) A 75 year old man presented with an ulcerated lesion on his leg, which he said had been present for
several months but now had increased in size and was bleeding. The lesion was surgically removed.
Specimen No. 1
Describe the lesion

What would be your differential diagnosis?

A representative section is shown (slide 288), list the features seen.

What would your diagnosis be now?

b) A 35 year old woman presents with a mole on her shoulder which has recently increased in size and
started to bleed. On examination there is a pigmented inflamed nodular lesion (diameter 1.2cm) with
irregular borders and two satellite nodules. The GP has taken a diagnostic incisional biopsy under local
anaesthetic for histology. Do you agree with the management so far?

The slide (308) is a representative section of the lesion, describe what you see.

What is the diagnosis?

From a prognostic point of view what are the important histological features?

She returns after 3 months with black urine, jaundice and convulsions. Admission to hospital is
arranged but she has a cardiac arrhythmia and a fatal cardiac arrest. An autopsy is done. How does the
findings in the brain (specimen no. 2) explain the signs and symptoms described?
2013 Case Study Questions

CASE 2
Name the two types of bone marrow?

Specimen No. 3

What condition is caused by this lesion?


Would the calcium level be increased or decreased?
Explain the mechanism.

What other clinical manifestations may this disease present with?

A 20 year old patient presents with fatigue, loss of weight and vague abdominal pains. On examination
he is noted to be pale. These are his blood results:

Haemoglobin (g/dL) 7.0 (14-18)


Erythrocytes (x10 perL) 2.2 (4.6-6)
Haematocrit 21 (42-50)
MCV (fL) 130 (80-95)
MCHC (%) 34 (32-35)
Reticulocytes (x10 perL) 41 (20-70)

Comment

A biopsy is taken from his ileum. The following slide (274) is from this biopsy. What abnormalities
are seen?

How can we connect the biopsy findings with the blood results?

Would a peripheral blood film have helped?


2013 Case Study Questions

CASE 3
Note: More than one answer may be correct.
The germinal centre of a lymph node:

A. Contains mainly T-lymphocytes. -


B. Contains dendritic reticulum cells. -
C. Contains cords and sinuses. -
D. Generates immunoglobulin-producing plasma cells. -
E. Is enlarged in a reactive lymph node. -

Hodgkins disease

A. Histologically characterized by the Reed-Sternberg cell. -


B. May be staged using the Ann Arbor system. -
C. Usually presents with painful lymphadenopathy. -
D. May be associated with eosinophillia. -
E. Associated with Askanazy cells. -

Burkitt’s lymphoma

A. Runs a prolonged couses if untreated. -


B. Is a tumour of T-lymphocytes. -
C. Is associated with the Ebstein-Barr virus. -
D. Has a geographical distributions which is similar to that of falciparum malaria. -
E. Rarely affects the jaw. -

Specimen No. 4

This is a specimen from a 25 year old patient who is HIV positive.


What is the likely diagnosis?

Would this be of T or B cell origin?

How aggressive is this disease likely to be?

What are B symptoms and is this patient likely to be suffering from them?
2013 Case Study Questions

CASE 4
A 70 year old man presents to the ER with shortness of breath and feeling faint. He gives a history of
chest pain for the past few days, which he thought was indigestion. He is a known hypertensive and
smokes 10-15 cigarettes per day. On examination he has a rapid weak pulse and muted heart sounds.
Unfortunately before any further tests he dies.
A post mortem is done.

Specimen No. 5 -

Describe the pathological events that led to this outcome.

Slide # 310 is a section of myocardium, what are the pathological features seen?

How old is this infarct?

Name the two patterns of infarction which one is more common?

What determines the extent of the infarct?

If this was due to atheroma what are the sequence of events leading to occlusion of the vessel?

What is reperfusion injury and why does it occur?

List 4 complications of myocardial infarction.


2013 Case Study Questions

CASE 5
A 27 year old female patient presents to her GP one week after a tooth extraction. She complains that
she feels tired and has been having recurrent fevers. On examination she is febrile, peripheral oedema
and her pulse is rapid and thready. On auscultation to the heart a mitral stenosis murmur is heard. She
is immediately sent to the ER.

Further history taking reveals that she had rheumatic fever as a child.

Urgent investigations are carried out including a chest Xray. Unfortunately she passes away soon after
these are taken.

Describe the slide #161, specimen No. 6 and chest Xray. Outline the disease mechanisms that have
occurred starting with the illness as a child.
• Slide #161 –

• Specimen No. 6 –

• X-ray -

What is the most common organism involved in this disease process?

The other group of valve vegetations are noninfected. Non-bacterial endocarditis falls into this
category, what is your understanding of this entity and give an example of when it may occur.
2013 Case Study Questions

CASE 6
A 78 year old man was admitted to hospital with fever, malaise and shortness of breath. A history of
chronic alcohol abuse was noted. On examination he was pyrexial, dehydrated and had dullness in the
right upper chest on percussion. Auscultation of the chest revealed diminished breath sounds on the
rights side and a pleural friction rub. Describe the Xray findings.

What factors may have contributed to this.

Slide 105 corresponds with this condition. Comment on the appearance and stage of inflammatory
response.

List three possible complications.

Describe Specimen Nos. 7 and 8

What are the likely diagnoses? Describe the classical histological appearance of this disease.

Name the organism involved. Outline the pathways and spectrum of tuberculosis from primary to
miliary tuberculosis.
2013 Case Study Questions

CASE 7
Describe the lesion seen in this X-ray (A). What is the most likely diagnosis and what would be the
histological appearance be?

Describe the lesion seen in Xray B and lesion seen in the specimen no. 9

What is the most likely diagnosis and why?

Describe the possible neurological complications.

Define paraneoplastic syndrome. List three that may occur in lung cancers.
2013 Case Study Questions

CASE 8
A 68 year old male visits his GP complaining of indigestion. On examination he has epigastric
tenderness. The doctor suspects that the patient has a peptic ulcer and prescribes cimetidine and
advises the patient to stop smoking. Two weeks later the patient has an episode of haematemesis and is
admitted to hospital. His pulse on admission is 108/min and BP 100/65. What could the possible
causes of the haematemesis be?

Other findings on examination were ascites, spider naevi and mild gynaecomastia. What is the likely
explanation of the haemetemesis?

What pathological processes are responsible for the ascites, varices, spider naevi and gynaecomastia?

His condition stabilizes and a liver biopsy is suggested. What should be checked first?

Slide 112 is from the biopsy. Describe the features seen and do these findings suggest a cause.

Specimen no. 10 is demonstrative of one of the complications of this disease, describe the lesion seen
and suggest the likely diagnosis.
2013 Case Study Questions

CASE 9
Draw and label a representative diagram of the features seen in slide 108. Is this likely to be acute or
chronic and why?

What organism is often implicated in this condition?

How do you determine whether to call a lesion an erosion or ulcer?

When do Curling and Cushing ulcers occur?

List two complications of peptic ulcer disease.

Describe the lesion seen in this specimen no. 11. What name is given to this lesion.

How do gastric adenocarcinomas spread and what is a Krukenberg tumour?


2013 Case Study Questions

CASE 10
What condition is illustrated by slide 205?

Why is it important to diagnose this condition

Diagnose the condition illustrated by specimen no. 12.

Give a definition that describes the lesion.

What two factors are important in the pathogenesis of this condition?

Note: More than one answer may be correct.


The following are bacterial causes of intestinal infection:
A. Giardia lamblia –
B. Balantidium coli -
C. Schistosoma mansoni -
D. Escherichia coli -
E. Histoplasma capsulatum -

Hirschsprung’s disease:
A. usually affects the right side of the colon -
B. is caused by failure of migration of neural crest cells -
C. is not associated with meconium ileus -
D. the abnormality is in the dilated portion of bowel -
E. may cause constipation –

The following polyps in the large bowel are precancerous:


A. tubular adenoma -
B. inflammatory polyp -
C. metaplastic polyp -
D. lymphoid polyposis -
E. villous adenoma -

You might also like